*NURSING > QUESTIONS & ANSWERS > NURSE MED SURG Cardiovascular | Chapters 33, 34 & 35 | Questions, Answers and Rationale and the Tes (All)

NURSE MED SURG Cardiovascular | Chapters 33, 34 & 35 | Questions, Answers and Rationale and the Test-Taking Tips.

Document Content and Description Below

The patient presents to the emergency department following a harrowing motor vehicle accident for evaluation despite having no injuries. The nurse anticipates which changes in the patient’s heart fu... nction? Select all that apply. < Increased heart rate (HR) Decreased stroke volume (SV) Decreased cardiac output (CO) Decreased blood pressure (BP) Increased myocardial contractility pp. 660-646 Which statement describes a thallium imaging scan? < This test helps to differentiate normal from diseased myocardium. This test helps to evaluate cardiac motion and calculate ejection fraction. This test helps to detect the location and size of acute myocardial infarctions. This test helps to detect the location and extent of acute or chronic myocardial infarctions. p. 661 A patient diagnosed with dehydration has a rhythm change that is reported to the nurse. After reviewing the rhythm below, what actions by the nurse are priorities? Select all that apply. <p>A patient diagnosed with dehydration has a rhythm change that is reported to the nurse. After reviewing the rhythm below, what actions by the nurse are <b>priorities</b>? < Evaluate urinary output Gently begin carotid massage Increase the intravenous fluids Assess the mucous membranes Prepare the patient for transcutaneous pacing Test-Taking Tip: If you can eliminate any responses as incorrect based on your knowledge, you will not be guessing randomly but will be exercising "informed guessing." p. 682 The nurse notices this rhythm on a patient's cardiac monitor. Which action should the nurse take first? < Administer oxygen Prepare for defibrillation Begin cardiopulmonary resuscitation Assess the patient's airway, breathing, and circulation p. 683 Determine the heart rate for a patient with 12 QRS complexes in the 6-second strip. < 12 100 110 120 (10x12=120bmp) Test-Taking Tip: Avoid looking for an answer pattern or code. There may be times when four or five consecutive questions have the same letter or number for the correct answer. p. 654 What symptoms are associated with chronic constrictive pericarditis? Select all that apply. < Dyspnea Dependent edema Crackles in the lungs Muffled heart sounds Jugular vein distention p. 713 Which symptoms indicate a possible dysrhythmia in a patient with dilated cardiomyopathy? Select all that apply. < Syncope Dyspnea Dizziness Palpitations Cardiomegaly p. 715 Which patient is best to assign to an LPN/LVN working on the telemetry unit? < Patient with heart failure who is receiving dobutamine Patient with rheumatic fever who has a new systolic murmur Patient with restrictive cardiomyopathy who uses oxygen for exertional dyspnea Patient with pericarditis who has a paradoxical pulse and distended jugular veins assign to an LPN/LVN working on the telemetry unit?</p> • Correct (484) Which statement is true regarding the anatomy of the heart? <p>Which statement is true regarding the anatomy of the heart?</p> Left ventricle (LV) collects oxygenated blood from the pulmonary veins. Left atrium (LA) receives deoxygenated blood from the superior and inferior vena cavae and coronary sinus. Right ventricle (RV) receives blood from the right atrium (RA) and propels it through the pulmonary artery and lungs. Right atrium (RA) receives blood from the left atrium (LA) through the aorta to circulate systemically through the body. p. 657 The head nurse is teaching a group of student nurses about cardiac valves. Which statement by the student nurse requires correction? <p>The head nurse is teaching a group of student nurses about cardiac valves. Which statement by the student nurse requires correction?</p> "The aortic valve separates the left ventricle (LV) from the aorta." "The tricuspid valve separates the right atria (RA) from the right ventricle (RV)." "The pulmonic valve separates the right ventricle (RV) from the pulmonary vein." "The mitral (bicuspid) valve separates the left atria (LA) from the left ventricle (LV)." p. 657 What is the average pressure of blood in the right atrium (RA)? <p>What is the average pressure of blood in the right atrium (RA)?</p> 0 to 5 mm Hg 20 to 50 mm Hg 60 to 70 mm Hg 100 to 120 mm Hg p. 657 Which portion of the heart is supplied by the right coronary artery (RCA)? <p>Which portion of the heart is supplied by the right coronary artery (RCA)?</p> Papillary muscle Ventricular septum Chordae tendineae Inferior portion of left ventricle (LV) p. 658 The nursing instructor is teaching a group of student nurses about the structure of the heart. Which statement does the nursing student know to be true? <p>The nursing instructor is teaching a group of student nurses about the structure of the heart. Which statement does the nursing student know to be true?</p> The heart is protected by a covering called the epicardium. The heart is separated into three halves by a muscular wall or septum. The heart pumps about 120 mL blood or 10 L/min in each beat during strenuous activity. The heart is a fist-sized muscular organ located in the mediastinum between the lungs. p. 657 A patient experiences a sudden, intense stabbing chest pain early in the morning while at rest. Which term best describes this condition? Anxiety Myocardial infarction Pleuropulmonary dysfunction Esophageal-gastric dysfunction p. 650 The left circumflex branch (LCX) of left main artery sometimes supplies blood to the portions of the interventricular septum. Which other portions of the heart receive blood from this branch? Select all that apply. Sinoatrial node Left atrium (LA) Right atrium (RA) Right ventricle (RV) Atrioventricular (AV) node Test-Taking Tip: Be alert for details about what you are being asked to do. In this Question Type, you are asked to select all options that apply to a given situation or patient. All options likely relate to the situation, but only some of the options may relate directly to the situation. pp. 657-643 A student nurse is discussing the characteristics of cardiac muscle cells with other students. Which characteristic of cardiac cell muscles is least likely to be important to the discussion? < Elasticity Contractility Automaticity Refractoriness While caring for a patient with cardiovascular disease (CVD), the nurse recognizes the patient is at risk for decreased cardiac output (CO). Which finding supports the nurse’s observation? <p>While caring for a patient with cardiovascular disease (CVD), the nurse recognizes the patient is at risk for decreased cardiac output (CO). Which finding supports the nurse&#x2019;s observation?</p> Cardiac output (CO) of 3 L/min Cardiac output (CO) of 5 L/min Cardiac output (CO) of 6 L/min Cardiac output (CO) of 8 L/min Test-Taking Tip: You have at least a 25% chance of selecting the correct response in multiple-choice items. If you are uncertain about a question, eliminate the choices that you believe are wrong and then call on your knowledge, skills, and abilities to choose from the remaining responses. p. 660 The head nurse is teaching the newly recruited nurses regarding the properties of the heart. Which term would she use that best describes the amount of blood ejected by the LV with each cardiac contraction? < Preload Cardiac index Heart rate (HR) Stroke volume (SV) p. 660 While monitoring the hemodynamic status of a patient with cardiovascular disease (CVD), the nurse finds that the patient’s cardiac output (CO) is 2.4 L/min. Which treatment strategy does the nurse expect from the primary health care provider in this situation? < Diuretics Beta blockers Positive inotropic drugs Calcium channel blockers Test-Taking Tip: Identifying content and what is being asked about that content is critical to your choosing the correct response. Be alert for words in the stem of the item that are the same or similar in nature to those in one or two of the options. p. 660 The nurse uses an arterial line in a patient suspected of cardiovascular disease (CVD) to obtain continuous, direct blood pressure (BP) readings and to obtain samples for arterial blood gas measurements. How does the nurse recognize that the patient is at risk for insufficient perfusion of body organs? < Mean arterial pressure (MAP) of 58 mm Hg Mean arterial pressure (MAP) of 62 mm Hg Mean arterial pressure (MAP) of 68 mm Hg Mean arterial pressure (MAP) of 70 mm Hg p. 659 Which statement describes Starling’s law of the heart? < The heart contracts more forcefully when it is filled more during diastole. The heart contracts more forcefully when it is emptied more during systole. The heart contracts more forcefully when coronary vascular resistance is minimized. The heart contracts more forcefully when total systemic vascular resistance is minimized. Rationale According to Starling’s law of the heart, the heart contracts more forcefully when it is filled more during diastole. The following statements are not consistent with Starling’s law: The heart contracts more forcefully when it is emptied more during systole; the heart contracts more forcefully when coronary vascular resistance is minimized; the heart contracts more forcefully when total systemic vascular resistance is minimized. p. 660 Which factor is least related to impedance? Blood viscosity Aortic compliance Coronary vascular resistance Total systemic vascular resistance The nursing instructor is teaching a group of student nurses about purposes of the vascular system. Which statement made by the student nurse requires correction? < "It returns blood to the heart for recirculation." "It carries cellular wastes to the excretory organs." "It allows lymphatic flow to drain tissue fluid back into circulation." "It helps in the transport of cellular wastes to excretory organs by the veins." Test-Taking Tip: Multiple-choice questions can be challenging, because students think that they will recognize the right answer when they see it or that the right answer will somehow stand out from the other choices. This is a dangerous misconception. The more carefully the question is constructed, the more each of the choices will seem like the correct response. p. 660 A registered nurse asks a student nurse to state an influencing factor that increases myocardial contractility. Which factor provided by the student nurse indicates effective learning? < Hypoxia Acidemia Sympathetic stimulation Parasympathetic stimulation p. 660 The registered nurse notes that when his patients are dehydrated, it is more difficult to gain peripheral intravenous access. He attributes this change to actions by which receptors present in the right atrium that are sensitive to pressure or volume changes, stimulate the sympathetic nervous system, and constrict the peripheral blood vessels upon stimulation? < Baroreceptors Stretch receptors Central chemoreceptors Peripheral chemoreceptors p. 660 Which patient’s condition may result in increased blood pressure (BP) via stimulation of central chemoreceptors? <p>Which patient&#x2019;s condition may result in increased blood pressure (BP) via stimulation of central chemoreceptors?</p> Hypoxia Hypoxemia Hypercapnia Hypovolemia Test-Taking Tip: Various receptors that are stimulated in response to a patient’s condition are given. Recall the condition that stimulates central chemoreceptors to regulate or maintain blood pressure (BP). p. 661 The nursing instructor is teaching nursing students about central chemoreceptors. Which statement, if made by the student, indicates that the teaching was effective? <p>The nursing instructor is teaching nursing students about central chemoreceptors. Which statement, if made by the student, indicates that the teaching was effective?</p> "They are sensitive primarily to hypoxemia." "They are stimulated when arterial walls stretch." "They are stimulated by hypercapnia and acidosis." "They are sensitive to pressure or volume changes." p. 661 Which factors influence the activity of the cardiovascular (CV) system? Select all that apply. <p>Which factors influence the activity of the cardiovascular (CV) system? <b>Select all that apply.</b> </p> Blood viscosity Physical activity Body temperature Aortic compliance Emotional behaviors p. 661 The nurse is reviewing the medical records of four older patients with changes in their cardiovascular system (CV). Which patient’s cardiovascular anomaly requires assessment of electrocardiogram (ECG) for a widening QRS complex and a longer QT interval? < Patient 1 Patient 2 Patient 3 Patient 4 p. 647 The nurse is caring for an older patient diagnosed with cardiovascular disease (CVD). After teaching the patient to change positions slowly, the nurse assesses the patient’s blood pressure (BP) and finds a drop in BP by 20 mm Hg when changing from a sitting to standing position. What cardiovascular changes related to aging does the nurse suspect the patient has? < Ineffective baroreceptors Enlarged left ventricle (LV) Calcification in mitral and aortic valves Thick, stiff, and less distensible arteries p. 647 An older patient may be at risk for developing left ventricular hypertrophy as a result of arterial changes related to aging. Which interventions should the nurse implement while caring for this patient? Select all that apply. < Assess the heart sounds for murmurs. Assess for activity intolerance and shortness of breath. Note increase in systolic, diastolic, and pulse pressures. Assess for dizziness when the patient changes positions. Assess electrocardiogram (ECG) for widening QRS complex and a longer QT interval. Test-Taking Tip: Read the question carefully before looking at the answers: (1) determine what the question is really asking; look for key words; (2) read each answer thoroughly and see if it completely covers the material asked by the question; (3) narrow the choices by immediately eliminating answers you know are incorrect. p. 647 Fill in the blankThe nurse finds that a patient’s stroke volume (SV) is approximately 80 mL at rest and heart rate (HR) is 70 beats/min. What is the patient’s cardiac output (CO)? 5.6 L/min Rationale Cardiac output (CO) is defined as heart rate (HR) x stroke volume (SV). Therefore, 80 mL/beat x 70 beats/min = 5600 mL/min. This patient's CO is 5.6 L/min. p. 660 Fill in the blankA patient has a cardiac output (CO) of 6.1 L/min and a body surface area of 2.3 m2. What is the cardiac index of this patient? Record your answer using two decimal places. L/min/m2 2.65 p. 660 Which cardiac parameter refers to the degree of myocardial fiber stretch at the end of diastole and is determined by the amount of blood returning from the venous system to the right side of the heart and the pulmonary system to the left side of the heart? < Preload Afterload Impedance Stroke volume (SV) p. 660 Which change in the cardiovascular system (CV) results in decreased stroke volume (SV), ejection fraction, and cardiac output (CO) during exercise? < Decrease in pacemaker cells Calcification in aortic and mitral valves Increase in the size of the left ventricle (LV) Thickening of the aorta and other large arteries p. 647 What happens when pacemaker cells decrease in number? <p>What happens when pacemaker cells decrease in number?</p> Hypertension may occur. Murmurs may be detected. Orthostatic or postural changes may be detected. Cardiac dysrhythmias and heart rates less than 60 beats/min may occur. p. 647 Which receptors are stimulated by hypoxemia? Baroreceptors Stretch receptors Central chemoreceptors Peripheral chemoreceptors p. 660 The registered nurse is teaching the student nurse about the variables influencing the stroke volume (SV). The registered nurse says, "It is the pressure or resistance the ventricles overcome to eject blood through the semilunar valves and into the peripheral blood vessels." Which variable influencing SV is the registered nurse referring to? <p>The registered nurse is teaching the student nurse about the variables influencing the stroke volume (SV). The registered nurse says, "It is the pressure or resistance the ventricles overcome to eject blood through the semilunar valves and into the peripheral blood vessels." Which variable influencing SV is the registered nurse referring to?</p> Preload Afterload Contractility Heart rate (HR) p. 660 Which class of medications maximizes cardiac performance in patients with heart failure by increasing the ventricular contractility? <p>Which class of medications maximizes cardiac performance in patients with heart failure by increasing the ventricular contractility?</p> Diuretics Beta blockers Inotropic agents Calcium channel blockers p. 660 Which physical assessment data does a nurse require to calculate a patient’s cardiac index? Select all that apply. <p>Which physical assessment data does a nurse require to calculate a patient&#x2019;s cardiac index? <b>Select all that apply.</b> </p> Age Height Weight Gender Waist size p. 660 What is the most appropriate nursing intervention for an older adult who is suspected of having decreased sensitivity in baroreceptors? Assessing the patient’s activity intolerance Teaching the patient to change positions slowly Questioning the patient about breathing difficulties Assessing the patient’s heart rate (HR) at rest and with activity p. 647 The nurse is caring for patient diagnosed with a myocardial infarction. The nurse knows which artery, if blocked, would lead to death of tissue in the anterior wall of the heart? < Circumflex artery Right coronary artery Left anterior descending artery Posterior descending coronary artery p. 659 Which cardiac valve maintains forward flow of blood through the heart by separating the left atrium (LA) from the left ventricle (LV)? < Aortic valve Tricuspid valve Pulmonic valve Bicuspid (mitral) valve p. 657 Ventricular changes Valvular abnormalities Orthostatic and postprandial changes Premature ventricular contractions (PVCs) p. 647 The nurse identifies the following conditions related to aging in four older patients. Which patient’s condition is associated with the changes in the conduction system? Patient 1 Patient 2 Patient 3 Patient 4 p. 647 The nurse is reviewing the results of cardiac output (CO) as well as the resistance in the arterioles to determine a cardiac component. Which component would the nurse determine? <p>The nurse is reviewing the results of cardiac output (CO) as well as the resistance in the arterioles to determine a cardiac component. Which component would the nurse determine?</p> Cardiac index Heart rate (HR) Stroke volume (SV) Blood pressure (BP) p. 660 The nurse is closely monitoring her patient’s vital signs and knows that which mean arterial pressure (MAP) is necessary to maintain adequate blood flow through the coronary arteries? <p>The nurse is closely monitoring her patient&#x2019;s vital signs and knows that which mean arterial pressure (MAP) is necessary to maintain adequate blood flow through the coronary arteries?</p> 55 mm Hg 60 mm Hg 65 mm Hg 70 mm Hg p. 659 Which statement regarding atrioventricular (AV) valves and semilunar valves of the heart is true? Atrioventricular (AV) valves prevent the back flow of blood into the atria during diastole. Atrioventricular (AV) valves allow the flow of blood from atria to ventricles during systole. Semilunar valves prevent blood from flowing back into the ventricles during diastole. Semilunar valves allow the coronary artery blood flow to the myocardium during systole. p. 657 The imaging studies of an older patient reveal an increase in the fibrous tissue and fat in the sinoatrial (SA) node. The nurse also finds that the patient’s heart rate (HR) is 50 beats/min. Which component of the changes in the cardiovascular system (CV) is associated with bradycardia and includes the SA node related to such complications? < Baroreceptors Cardiac valves Left ventricle (LV) Conduction system p. 647 A nurse assesses the heart rate (HR) and rhythm of an older patient who is suspected of having age-related cardiac valve dysfunction. Which is the most appropriate nursing intervention that would help in confirming this in the patient? < Asking about the patient’s drug history Asking about the patient’s medical history Assessing if the patient has an abnormal gait Assessing if the patient has difficulty breathing p. 647 Which term is used to refer to the degree of myocardial fiber stretch at the end of diastole and just before contraction? < Preload Afterload Impedance Stroke volume (SV) p. 660 Which body system does the cardiovascular (CV) system work in conjunction with to meet the human needs for oxygenation and tissue perfusion? < Renal Neurologic Respiratory Integumentary p. 656 While performing a cardiovascular assessment of a patient, the nurse detects a very loud heart murmur accompanied by a palpable thrill. This is audible even if the stethoscope is partially off the patient's chest. What grade of heart murmur does this describe? < Grade V Grade III Grade IV Grade VI p. 655 A patient reports fatigue, palpitations, and dyspnea after performing an ordinary task. According to the New York Heart Association's functional classification of cardiovascular disability, in which class does the patient belong? Class I Class II Class III Class IV p. 651 Which is a physiologic change of the cardiovascular system associated with aging? <p>Which is a physiologic change of the cardiovascular system associated with aging?</p> Increase in the sensitivity of baroreceptors Decrease in the number of pacemaker cells Thinning of the aorta and other larger arteries Reduction in size and efficacy of the left ventricle p. 647 The nurse is caring for a patient taking amlodipine for hypertension. For which side effect is the nurse regularly assessing? <p>The nurse is caring for a patient taking amlodipine for hypertension. For which side effect is the nurse regularly assessing?</p> Paradoxical blood pressure Central and peripheral cyanosis Postural orthostatic hypotension Dependent foot and ankle edema p. 655 The nurse is caring for a patient with an arterial line. How does the nurse recognize that the patient is at risk for insufficient perfusion of body organs? <p>The nurse is caring for a patient with an arterial line. How does the nurse recognize that the patient is at risk for insufficient perfusion of body organs?</p> Po 2 is reported as 78 mm Hg. Right atrial pressure is 4 mm Hg. Mean arterial pressure (MAP) is 58 mm Hg. Pulmonary artery wedge pressure (PAWP) is 7 mm Hg. p. 657 The nurse in a coronary care unit interprets information from hemodynamic monitoring. The patient has a cardiac output of 2.4 L/min. Which action would be taken by the nurse? <p>The nurse in a coronary care unit interprets information from hemodynamic monitoring. The patient has a cardiac output of 2.4 L/min. Which action would be taken by the nurse?</p> Administer a stat dose of metoprolol. Ask the patient to perform the Valsalva maneuver. No intervention is needed; this is a normal level. Collaborate with the health care provider to administer a positive inotropic agent. p. 660 Which statement about blood pressure (BP) is correct? <p>Which statement about blood pressure (BP) is correct?</p> Kidneys help to regulate cardiovascular activity. Blood pressure rises when body temperature decreases. Sodium and water is retained when renal blood flow increases. Anger causes the sympathetic nervous system to decrease BP and heart rate. p. 661 How does the nurse identify a bruit during a cardiac assessment? <p>How does the nurse identify a bruit during a cardiac assessment?</p> Presence of a weak pulse Absence of pulse sounds Presence of a bounding pulse Presence of swishing sounds p. 653 Cardiac output is represented by which formula? Heart rate × stroke volume Systolic – diastolic blood pressure Systolic pressure ÷ diastolic pressure Pulmonary artery diastolic = pulmonary artery wedge pressure Rationale Cardiac output = heart rate × stroke volume. Systolic – diastolic blood pressure = pulse pressure. Under normal conditions, the pulmonary artery diastolic pressure theoretically is equivalent to the pulmonary capillary wedge pressure, but that does not answer the question. Systolic pressure ÷ diastolic pressure is an oversimplified equation for the mean arterial blood pressure. p. 660 Which is a modifiable risk factor for coronary heart disease? <p>Which is a modifiable risk factor for coronary heart disease?</p> Age Sex Hypertension Family history p. 661 The nurse is assessing a patient admitted to the cardiac care unit. The nurse would anticipate reduced pulse pressure in the patient with which condition? Hypertension Mitral stenosis Atherosclerosis Aortic regurgitation p. 652 The patient presents to the emergency department with chest pain and is diagnosed with angina pectoris. The nurse prepares to administer which medication? Nitrates Opioids Analgesics Antipyretics p. 651 The nurse is caring for a patient with mitral valve prolapse who asks about the purpose of the mitral valve. How should the nurse respond? < "It prevents blood from flowing from the right atrium into the right ventricle during systole." "Located between the left ventricle and the aorta, it promotes the flow of blood to the entire body." "Located between the right atrium and right ventricle, it prevents the flow of blood between these chambers." "Located between the left atrium and left ventricle, it prevents blood from flowing back into the atrium during systole." p. 657 Which disease or occurrence is the leading cause of death in adults? <p>Which disease or occurrence is the <b>leading</b> cause of death in adults?</p> Cancers Accidents Heart disease Chronic obstructive pulmonary disease p. 657 During a screening test for asymptomatic coronary artery disease, the patient is assessed for cardiovascular response to an increased workload. What instruction should the nurse provide after the test? "Consume a light meal 2 hours after the test." "Refrain from taking a hot shower for 2 hours after the test." "Refrain from drinking alcohol or caffeinated beverages after the test." "Withhold cardiovascular medications such as beta blockers after the test." p. 660 During a diagnostic test for a patient suspected of having a cardiac tumor, the nurse elevates the patient’s head to 20 degrees and places a small transducer on the patient’s chest at the level of fourth intercostal space near the left sternal border. Which diagnostic test is the patient undergoing? Echocardiography Electrocardiography Magnetic resonance imaging Myocardial nuclear perfusion imaging pp. 660-661 The nurse assesses peripheral pulses and auscultates the patient’s heart and lungs before a cardiac catheterization test for a patient suspected to have valvular dysfunction. Which other intervention should the nurse implement before the procedure? Instruct the patient to withhold any medication for diuretics therapy. Administer steroids if the patient has contrast-induced renal toxicity. Administer acetylcysteine if the patient has an allergy to iodine-based contrast agents. Instruct the patient to withhold medication if the patient is on calcium channel blockers. Test-Taking Tip: Make certain that the answer you select is reasonable and obtainable under ordinary circumstances and that the action can be carried out in the given situation. p. 658 A patient reports to the nurse, "I have a sharp, stabbing pain that usually spreads to my left side or my back and is relieved when I sit upright. It tends to come and go." How should the nurse interpret this concern? < The patient has angina. The patient has pericarditis. The patient has valvular dysfunction. The patient has myocardial infarction. p. 649 The nurse finds that the patient’s anginal syndrome is present even at rest and the discomfort is increased if any physical activity is undertaken. According to New York Heart Association Functional Classification of cardiovascular disability, to which class does the patient belong? Class I Class II Class III Class IV p. 651 When performing a cardiovascular physical assessment, which finding indicates the patient is experiencing chronic hypoxemia? Peripheral pitting edema Clubbing of the fingers and toes Decreased skin turgor over the chest Petechiae on the arms and inner thigh STUDY TIP: You have a great resource in your classmates. We all have different learning styles, strengths, and perspectives on the material. Participating in a study group can be a valuable addition to your nursing school experience. p. 652 Which statement reflects correct education for a patient with a blood pressure (BP) of 136/86 mm Hg? It is a normal reading, so no action is needed. The patient has hypertension or high BP. The BP is low and the patient should be further assessed for orthostatic hypotension. Lifestyle modification is needed to decrease the workload on the heart. p. 647 The nurse is caring for a patient admitted with pericarditis. Which symptom does the nurse anticipate the patient will report? < Viselike substernal chest pain in response to exertion Substernal chest pain described as sharp and stabbing Dizziness requiring the patient to stop activities and rest Abrupt awakening with dyspnea which worsens after sitting on the edge of the bed Test-Taking Tip: If the question asks for an immediate action or response, all of the answers may be correct, so base your selection on identified priorities for action. p. 650 A patient with heart failure reports a 7.6-lb weight gain in the past week. What intervention does the nurse anticipate from the health care provider? <p>A patient with heart failure reports a 7.6-lb weight gain in the past week. What intervention does the nurse anticipate from the health care provider?</p> Dietary consult Restricted activity Sodium restriction Daily weight monitoring p. 650 The nurse is reviewing the laboratory results of different male patients in a health care setting suspected of having cardiovascular disorders. Which patient’s laboratory findings indicate a risk for coronary artery disease (CAD)? Patient 1 Patient 2 Patient 3 Patient 4 Test-Taking Tip: The total cholesterol, triglycerides, HDLs, and LDLs levels are mentioned. You need to recollect the normal ranges to determine which finding is abnormal. p. 656 A patient recovering from cardiac angiography develops slurred speech. What does the nurse do first? Calls in another nurse for a second opinion. Maintains NPO (nothing by mouth) until this resolves. Explains to the patient and family that this is expected after sedation. Performs a complete neurologic assessment and notifies the health care provider. Which laboratory findings are consistent with acute coronary syndrome (ACS)? Select all that apply. < Troponin 3.2 ng/mL Triglycerides 400 mg/dL Myoglobin of 234 mcg/L C-reactive protein 13 mg/dL Lipoprotein-a (Lp[a]) 18 mg/dL p. 656 The nurse is educating a group of women about the differences in symptoms of myocardial infarction (MI) in men versus those in women. Which information should be included? <p>The nurse is educating a group of women about the differences in symptoms of myocardial infarction (MI) in men versus those in women. Which information should be included?</p> Men do not tend to report chest pain. Men are more likely than women to die after MI. Men more than women tend to deny the importance of symptoms. Women may experience extreme fatigue and dizziness as sole symptoms. p. 648 A patient has been admitted to the hospital with chest pain radiating down the left arm. The pain has been unrelieved by rest and antacids. Which test result best confirms that the patient sustained a myocardial infarction? Troponin of 5.2 ng/mL C-reactive protein of 1 mg/dL Homocysteine level 13 mmol/L Creatine kinase (CK) of 125 mg/dL p. 656 The nurse is preparing the patient for an echocardiogram. Which point reflects correct patient teaching before the procedure? "You must not eat or drink anything for 8 hours before the test." "You will need to drink a contrast dye 1 hour before the procedure." "The technician will put lubricant on a transducer and rub this on your chest." "After the technician injects you with a radioisotope, you will be asked to exercise a few additional minutes." A patient on a telemetry unit is scheduled for electrophysiologic studies (EPS). Which condition does the patient likely have? Pericarditis Post–sudden cardiac arrest Allergy to intravenous contrast Episode of acute coronary syndrome Test-Taking Tip: Watch for grammatical inconsistencies. If one or more of the options is not grammatically consistent with the stem, the alert test taker can identify it as a probable incorrect option. When the stem is in the form of an incomplete sentence, each option should complete the sentence in a grammatically correct way. p. 660 A patient is brought to the emergency department for chest pain that began 30 minutes ago. Upon assessment, the patient says to the nurse, "The pain is gone now; it was a false alarm. I need to get back to mowing the lawn." Which response best describes the patient's statements? Anger Denial Bargaining Adaptation Test-Taking Tip: Look for answers that focus on the patient or are directed toward feelings. p. 655 A patient has a magnesium level of 1.0 mEq/L. Which complication of this electrolyte imbalance does the nurse anticipate? Diabetes Heart failure Myocardial infarction Ventricular dysrhythmia p. 657 The nurse anticipates finding a bounding or hyperkinetic pulse in the patient with which disorder or condition? Heart failure Thyrotoxicosis Carotid stenosis Syncope and dehydration STUDY TIP: When forming a study group, carefully select members for your group. Choose students who have abilities and motivation similar to your own. Look for students who have a different learning style than you. Exchange names, email addresses, and phone numbers. Plan a schedule for when and how often you will meet. Plan an agenda for each meeting. You may exchange lecture notes and discuss content for clarity or quiz one another on the material. You could also create your own practice tests or make flash cards that review key vocabulary terms. p. 653 The nurse is caring for patient who has just undergone cardiac catheterization. For which conditions is the nurse vigilantly assessing? Select all that apply. Hypovolemia Polycythemia Cardiac tamponade Fluid volume excess Reaction to contrast medium pp. 657-658 A 72-year-old patient admitted with fatigue and dyspnea has elevated white blood cell count, low-density lipoproteins, serum troponin I level, and C-reactive protein. Which finding is consistent with acute coronary syndrome (ACS) and should be communicated immediately to the health care provider? C-reactive protein Serum troponin I level Low-density lipoproteins White blood cell (WBC) count p. 656 A patient who is suffering dyspnea on exertion and congestive heart failure will likely report which symptom during the health history? <p>A patient who is suffering dyspnea on exertion and congestive heart failure will likely report which symptom during the health history?</p> Fatigue Slow heart rate Swelling of one leg Brown discoloration of lower extremities p. 649 A patient who is to undergo cardiac catheterization should be taught which essential information by the nurse? <p>A patient who is to undergo cardiac catheterization should be taught which essential information by the nurse?</p> "Keep your affected leg straight for 2 to 6 hours." "Monitor the pulses in your feet when you get home." "Do not take your blood pressure medications on the day of the procedure." "Take your oral hypoglycemic with a sip of water on the morning of the procedure." p. 659 The nurse is teaching a patient about the purpose of electrophysiology studies (EPS). Which statement by the nurse reflects the most correct teaching? "This test evaluates you for potentially fatal cardiac rhythms." "This is a noninvasive test performed to assess your heart rhythm." "This is a painless test that is done to assess the structure of your heart using sound waves." "You will receive an injection of dobutamine and will walk on a treadmill to reveal whether you have coronary artery disease." p. 659 Which patient should the charge nurse assign to a graduate RN who has completed 2 months of orientation to the coronary care unit? Patient with a new diagnosis of heart failure who needs a pulmonary artery catheter inserted Patient who has many questions about the electrophysiology studies (EPS) scheduled for today Patient who has just arrived after a coronary arteriogram and has vital signs requested every 15 minutes Patient with acute electrocardiographic changes who is requesting nitroglycerin for left anterior chest pain p. 657 Which action should the nurse delegate to experienced unlicensed assistive personnel (UAP) working in the cardiac catheterization laboratory? Assess preprocedure medications the patient took that day. Obtain patient vital signs and a resting electrocardiogram (ECG). Have the patient sign the consent form before the procedure is performed. Educate the patient about the need to remain on bedrest after the procedure. p. 659 An RN and an LPN/LVN, both of whom have several years of experience in the intensive care unit, are caring for a group of patients. Which patient is appropriate for the RN to assign to the LPN/LVN? A patient with pulmonary edema who requires hourly monitoring of pulmonary artery wedge pressures A patient with acute coronary syndrome who has just been admitted and needs an admission assessment A patient who has intermittent chest pain and requires teaching about myocardial nuclear perfusion imaging A patient who was admitted with peripheral vascular disease and needs assessment of the ankle-brachial index p. 653 The nurse is performing an admission assessment for a patient scheduled to undergo a coronary arteriogram. Which symptom, if reported to the nurse, would be most important to be passed on to the provider prior to the procedure? The patient develops wheezes and dyspnea after eating crab or lobster. The patient has had intermittent substernal chest pain for 6 months. The patient reports that a previous arteriogram was negative for coronary artery disease. The patient has peripheral vascular disease, and the dorsalis pedis pulses are difficult to palpate. p. 658 The nurse is assessing a patient with mitral stenosis who is to undergo a transesophageal echocardiogram (TEE) today. Which nursing action is essential? Validate that the patient has remained NPO. Reassure the patient that the test is painless. Teach the patient about the reason for the TEE. Auscultate the patient's precordium for murmurs. p. 660 Which statement by the patient with a recent cardiovascular diagnosis indicates maladaptive denial? "I don't think it is as bad as the doctors say." "I don't need to change. It hasn't killed me yet." "I will have to change my diet and exercise more." "I don't know how I am going to change my lifestyle." p. 655 Which statement about diagnostic cardiovascular testing is correct? <p>Which statement about diagnostic cardiovascular testing is correct?</p> The left side of the heart is catheterized first and may be the only side examined. Holter monitoring allows periodic recording of cardiac activity during an extended period of time. An alternative to injecting a medium into the coronary arteries is intravascular ultrasonography. Complications of coronary arteriography include stroke, nonlethal dysrhythmias, arterial bleeding, and thromboembolism. p. 659 Which patient has the highest risk for cardiovascular disease? Man who is sedentary and reports four episodes of strep throat Woman with abdominal obesity who exercises three times per week Man who smokes and whose father died at 49 from myocardial infarction (MI) Woman with diabetes whose high-density lipoprotein (HDL) cholesterol is 75 p. 646 Which symptomatology is most consistent with myocardial infarction (MI) requiring notification of the health care provider? Claudication and fatigue Abdominal pain and belching Sternal pressure and diaphoresis Dyspnea on exertion (DOE) and inability to sleep supine p. 649 Which statement reflects correct cardiac physical assessment technique? Assess for carotid bruit by auscultating over the anterior neck. Palpate the apical pulse over the third intercostal space in the mid-clavicular line. Auscultate the aortic valve in the second intercostal space at the right sternal border. Evaluate for orthostatic hypotension by moving the patient from a standing to a reclining position. p. 654 After a cardiac catheterization, the patient should increase fluid intake for which reason? The dye causes an osmotic diuresis. The dye contains a heavy sodium load. The pedal pulses will be more easily palpable. NPO status will cause the patient to be thirsty. p. 659 The nurse is reviewing the medical record of a patient admitted with heart failure. Which laboratory result warrants a call to the health care provider by the nurse for further instructions? Calcium 8.5 mEq/L Potassium 3.0 mEq/L Magnesium 2.1 mEq/L International Normalized Ratio (INR) of 1.0 p. 656 The nurse is assessing the cardiac health of a patient with a body mass index (BMI) of 30. What does the nurse tell the patient about maintaining a healthy weight? "Include high calorie, nutrient-rich foods." "Take a 30-minute brisk walk twice a week." "Include vegetables and dairy products in your diet." "Avoid eating more calories than you can burn every day." p. 648 The nurse is assessing a patient who reports an inability to sleep well at night due to difficulty breathing. The patient usually uses two pillows and at times requires an extra pillow to promote sleep. How does the nurse document this finding? Fatigue Orthopnea Dyspnea on exertion (DOE) Paroxysmal nocturnal dyspnea (PND) p. 649 The nurse is assessing a patient who reports a cramping sensation in the legs and buttocks after a walk. The sensation ceases after resting awhile. How does the nurse document this finding? < Syncope Palpitation Claudication Near-syncope p. 651 How does the nurse classify the patient with cardiovascular disease who experiences fatigue and palpitations with ordinary physical activity as per the New York Heart Association functional classification system? Class I Class II Class III Class IV p. 651 Which type of edema causes the nurse to suspect chronic venous insufficiency in a patient? Foot and ankle edema Abdominal and leg edema Bilateral edema of the legs Unilateral edema in the lower extremity p. 652 Which description is characteristic of peripheral cyanosis? Presence of rubor (dusky redness) in a dependent foot Presence of bluish discoloration of the nail beds and earlobes Presence of warmth and pink coloration in the palm and nail beds Presence of moisture, coolness, and pale coloration in the palms and nail beds p. 652 The nurse is caring for a patient after cardiac catheterization. What follow-up care should the nurse provide for this patient? < Apply a soft knee brace. Restrict oral and IV fluids. Remove the dressing after 6 hours. Encourage the patient to walk the next day. p. 659 A patient is scheduled for an exercise electrocardiography (stress test). What does the nurse tell the patient about preparing for the test? <p>A patient is scheduled for an exercise electrocardiography (stress test). What does the nurse tell the patient about preparing for the test?</p> "Take all prescribed medications before the test." "Have a light meal at least 6 hours before the test." "Exercise the day prior to the test to get good results." "Avoid caffeine-containing beverages on the day of the test." p. 659 The nurse is caring for a patient who has been ordered a magnetic resonance imaging (MRI) test. What does the nurse tell the patient about preparing for the test? <p>The nurse is caring for a patient who has been ordered a magnetic resonance imaging (MRI) test. What does the nurse tell the patient about preparing for the test?</p> "Avoid meals 6 hours before the test." "Hypertension may occur during the test." "Remove all jewelry, fasteners, and hair clips." "Avoid cardiovascular drugs on the day of the test." p. 661 A patient is admitted for chest pain. The nurse questions the patient about the use of which substance associated with heart disease? <p>A patient is admitted for chest pain. The nurse questions the patient about the use of which substance associated with heart disease?</p> Opiates Cocaine Marijuana Barbiturates p. 648 Which drug is used in cardiovascular assessment to simulate the effect of exercise in a patient who is unable to perform an exercise-tolerance test on a treadmill? Propranolol Dipyridamile Thallous chloride Fluoro-18-deoxyglucose p. 660 Which step is essential for the nurse to complete before the patient undergoes cardiac catheterization? Obtaining informed consent Administering an ordered diuretic Providing patient education about sensations during the procedure Assessing for allergies to iodine-containing substances and contrast media p. 658 Which signs and symptoms are seen with suspected pericarditis? Select all that apply. Squeezing, viselike chest pain Chest pain relieved by sitting upright Sudden-onset chest pain relieved by NSAIDs Chest and abdominal pain relieved by antacids Pain in the chest described as sharp or stabbing p. 650 Which of these factors contribute to the risk for cardiovascular disease? Select all that apply. Smoking Low blood pressure Consuming a diet rich in fiber Elevated C-reactive protein levels Elevated high-density lipoprotein (HDL) cholesterol level p. 656 The nurse is assessing a patient's smoking history. What does the nurse ask the patient who reports being a nonsmoker? Select all that apply. "Have you ever smoked?" "Does anybody in your home smoke?" "Do you smoke when you are stressed?" "Do you have smokers among your friends?" "Do you wake up from sleep with an urge to smoke?" p. 647 Which is true regarding anxiety-related chest discomfort? It worsens on inspiration. It lasts for at least 3 hours. It can be a dull ache or a sharp stabbing pain. It is sudden and begins without precipitating factors. p. 650 According to the New York Heart Association Functional Classification of Cardiovascular Disability, which class includes patients with symptoms of fatigue, palpitations, dyspnea, and angina even after ordinary physical activities? Class I Class II Class III Class IV p. 651 What are modifiable risk factors for cardiovascular disease (CVD)? Select all that apply. Male, age 35 years A patient's limited physical activity Family history of cardiovascular disease Body mass index (BMI) greater than 30 A patient's five pack-year smoking history p. 646 Which statement about fatigue is correct? It is referred to as difficult or labored breathing. It is referred to as a brief loss of consciousness. It is a feeling of tiredness or weariness resulting from activity. It is a feeling of fluttering or an unpleasant feeling in the chest caused by an irregular heartbeat. p. 650 Which noncardiac factors may precipitate palpitations? Select all that apply. Fatigue Insomnia Hyperthyroidism Sinus tachycardia Paroxysmal supraventricular tachycardia p. 650 What type of patient does class III of the New York Heart Association Functional Classification system include? Patients with cardiac disease but without restricted physical activity Patients with cardiac disease resulting in little limitation of physical activity Patients with cardiac disease that may result in a noticeable limitation of physical activity Patients with cardiac disease resulting in a lack of ability to perform any physical activity Test-Taking Tip: Identify option components as correct or incorrect. This may help you identify a wrong answer. p. 651 According to the New York Heart Association Functional Classification of Cardiovascular Disability, which class consists of patients with symptoms of cardiac insufficiency even at rest? Class I Class II Class III Class IV Rationale According to the New York Heart Association Functional Classification of Cardiovascular Disability, class IV consists of patients with symptoms of cardiac insufficiency even at rest. Classes I, II, and III consist of patients who remain comfortable at rest. p. 651 Which structure of the heart is responsible for receiving deoxygenated venous blood as well as the blood from the heart muscle? Septum Pericardium Right atrium Right ventricle What is the function of the semilunar valves? Maintaining the forward flow of blood Separating the atria from the ventricles Separating the left ventricle from the aorta Preventing blood from flowing back into the ventricles p. 643 The nurse is assessing a patient's heart and finds an abnormal observation with the "dub" sound of the patient's cardiac cycle. The nurse understands that which part of the cardiac cycle may need to be explored further? Ventricular filling Ventricular ejection Isovolumetric relaxation Isovolumetric contraction p. 644 The nurse is taking a patient's cardiac output (CO). What does the nurse assess? The number of times the ventricles contract each minute; the amount of blood ejected by the left ventricle during each contraction The total surface area of the body; the degree of myocardial fiber stretches at the end of diastole and just before contraction The degree of myocardial fiber stretches at the end of diastole and just before contraction; the number of times the ventricles contract each minute The amount of blood ejected by the left ventricle during each contraction; the pressure or resistance that the ventricles must overcome to eject blood through the semilunar valves and into the peripheral blood vessels p. 645 What quality and severity of pain is associated with angina? Squeezing, vise-like Intense stabbing, vise-like Sharp stabbing, moderate to severe Moderate ache, worse on inspiration p. 650 What changes in the cardiac system does the nurse take into consideration prior to counseling an older adult about exercise? A decreased stroke volume occurs. Chemoreceptors are less sensitive. Cardiac conduction time decreases. There is an increased ejection fraction. Test-Taking Tip: You have at least a 25% chance of selecting the correct response in multiple choice items. If you are uncertain about a question, eliminate the choices you believe are wrong, and then call on your knowledge, skills, and abilities to choose from the remaining responses. p. 647 What does the nurse anticipate finding during the assessment of an electrocardiogram (ECG) in an older adult? < Junction rhythms Atrial dysrhythmias Ventricular tachycardia Narrowed QRS complexes p. 647 Which property of the heart is considered its mechanical activity? < Excitability Conductivity Automaticity Contractility Test-Taking Tip: If the question asks for an immediate action or response, all of the answers may be correct, so base your selection on identified priorities for action. p. 676 What specialized function of myocardial cells gives them a pacing function? Excitability Contractility Conductivity Automaticity p. 676 Which statement is true about an atrial kick? <p>Which statement is true about an atrial kick?</p> It results in cardiac arrest It results in cardiac failure It results in a decreased heart rate It results in increased cardiac output p. 676 What is the primary pacemaker of the heart? <p>What is the primary pacemaker of the heart?</p> Bundle of His Purkinje fibers Sinoatrial (SA) node Atrioventricular (AV) node Test-Taking Tip: You have at least a 25% chance of selecting the correct response in multiple-choice items. If you are uncertain about a question, eliminate the choices that you believe are wrong and then call on your knowledge, skills, and abilities to choose from the remaining responses. p. 674 What part of the cardiac conduction system is responsible for ventricular depolarization? Purkinje fibers Sinoatrial (SA) node Transitional cell zone Atrioventricular (AV) node p. 676 What would happen if the sinoatrial (SA) node generates electrical impulses at a higher rate? The heart rate will increase The blood pressure will decrease The atrial depolarization will not occur The ventricular depolarization will not occur Test-Taking Tip: Sometimes the reading of a question in the middle or toward the end of an exam may trigger your mind with the answer or provide an important clue to an earlier question. p. 676 What is the function of T cells in the atrioventricular junction area? T cells cause the ventricles to contract The T cells help to speed up impulses to the ventricles The T cells generate impulses spontaneously and rhythmically The T cells cause a delay in the conduction of impulses to the atrioventricular (AV) node p. 676 What part of the conduction system is responsible for the contraction of ventricles? Purkinje fibers Sinoatrial (SA) node Transitional cell zone Atrioventricular (AV) node p. 676 Which waveform indicates proper function of the sinoatrial (SA) node? < The ST segment is elevated The QRS complex is present The PR interval is 0.24 second A P wave precedes every QRS complex p. 684 The nurse is providing cardiopulmonary resuscitation to a patient who was found in ventricular tachycardia with no pulse. The advanced cardiac life support (ACLS) team arrives. What is the nurse's priority responsibility at this time? Check on other assigned patients Provide comfort and support for the family Provide information about the patient to the ACLS team Replace the equipment used during cardiopulmonary resuscitation p. 686 What area of the cardiac conduction system is responsible for generating the electrical impulse in the heart? AV node Bundle of His Purkinje fibers The sinus node p. 676 The nurse is placing leads on a patient for a 12-lead electrocardiogram (ECG). Which lead acts as the ground electrode? < Chest lead Positive lead Right leg lead Precordial lead p. 678 The nurse is attempting to analyze an electrocardiogram (ECG) strip. How many large blocks represent a 6-second strip? 5 15 25 30 p. 684 Using calipers, the nurse has marked out the P to P intervals. The nurse recognizes a variation of two small blocks. Which action should the nurse take first? Contact the health care provider Document the rhythm as regular Verify that the leads are still attached to the patient Ask the patient if they would like PRN pain medication p. 670 The nurse is caring for a patient who develops torsades de pointes. What medication should the nurse administer? Digoxin Amiodarone Atropine sulfate Magnesium sulfate p. 686 The nurse is caring for a patient diagnosed with supraventricular tachycardia (SVT) with a nonsustained ventricular response. Which symptom does the nurse anticipate the patient to report? Syncope Weakness Shortness of breath Occasional palpitations p. 676 The nurse enters a patient's room and notices a patient is confused, diaphoretic, and has a pulse rate of 41 bpm. Which action should the nurse take first? Initiate chest compressions Assess the MAR for drugs that slow conduction Request informed consent for a transcutaneous pacing Prepare to administer 0.5 mg of atropine intravenously Test-Taking Tip: Key words or phrases in the question stem such as first, primary, early, or best are important. Similarly, words such as only, always, never, and all in the alternatives are frequently evidence of a wrong response. No real absolutes exist in life; however, every rule has its exceptions, so answer with care. p. 674 The nurse is educating a patient diagnosed with premature ventricular contractions (PVCs). The patient has been experiencing 1000-1500 PVCs in a 24-hour period. Which treatment education should be included in the teaching? Stress management Low potassium diet Beta-adrenergic blocking agent Battery replacement for a biventricular pacemaker p. 683 The nurse looks at the monitor and notices ventricular tachycardia. What action should the nurse take first? Call for help Assess the patient Begin cardiopulmonary resuscitation (CPR) Prepare to administer intravenous (IV) lidocaine Test-Taking Tip: Have confidence in your initial response to an item because it more than likely is the correct answer. p. 684 The nurse correctly identifies which medications as Class II antidysrhythmic used to decrease heart rate and conduction velocity? Select all that apply. Sotalol hydrochloride Esmolol hydrochloride Tocainide hydrochloride Propranolol hydrochloride Amiodarone hydrochloride Test-Taking Tip: Be alert for details about what you are being asked to do. In this Question Type, you are asked to select all options that apply to a given situation or patient. All options likely relate to the situation, but only some of the options may relate directly to the situation. p. 685 The nurse teaches a patient diagnosed with atrial fibrillation about verapamil hydrochloride, a new prescription the patient will be taking orally at home. Which statement by the patient indicates the teaching was successful? "I will need to check my pulse every 8 hours." "I should lay flat for an hour after I take the medicine." "I will bear down if I feel my heart starting to beat fast." "I will take my time getting up and changing positions." The nurse is caring for a patient 2 hours after surgery. The patient has been vomiting and reports feeling dizzy. The nurse evaluates the electrocardiogram (ECG) rhythm seen in the image below. Which action by the nurse is priority? Request an order for atropine Encourage deep breathing exercises Administer the prescribed antiemetic Teach the patient Valsalva maneuvers The emergency department nurse is caring for a healthy adult patient with supraventricular tachycardia (SVT). The nurse is preparing to administer adenosine and notices a normal sinus rhythm. What action by the nurse is appropriate? < Administer the medication Re-apply the leads of the ECG Document the change in rhythm Encourage the patient to bear down p. 678 The nurse is preparing to administer adenosine to a patient experiencing supraventricular tachycardia (SVT). The nurse should have what other items readily available? Select all that apply. Lamin AC Rivaroxaban Normal saline bolus Blood for transfusion Emergency equipment p. 678 What interventions should a nurse prepare when caring for a patient experiencing stable ventricular tachycardia? Select all that apply. Oxygen Atropine Adenosine Bi-ventricular pacing 12-lead electrocardiogram (ECG) The nurse is assigned the following four patients. Which patient should the nurse see first? A patient diagnosed with a bradydysrhythmia who has a pulse of 62 bpm at this time A patient admitted with chest pain in normal sinus rhythm who has no discernible P waves at this time A patient who presents uncontrolled atrial fibrillation and underwent a successful cardioversion yesterday A patient waiting for discharge who was in torsades de pointes when admitted, who is now in normal sinus rhythm p. 677 Which situation is appropriate for a nurse to perform synchronized countershock? Select all that apply. < Ventricular fibrillation Stable atrial tachydysrhythmias Ventricular asystole or standstill Premature ventricular contractions Unstable ventricular tachydysrhythmias The nurse is analyzing the electrocardiographic rhythm of a patient and identifies an abnormal T wave. What problems are potential causes for this abnormality? Select all that apply. < Anxiety Nicotine use Myocardial infarction Ventricular hypertrophy Chronic obstructive pulmonary disease (COPD) p. 671 Which finding on an electrocardiogram (ECG) strip is characterized as normal? PR interval of 0.25 seconds Ventricular rate of 105 bpm QRS duration of 0.05 seconds P waves present after QRS complex pp. 670-671 The nurse is caring for a patient diagnosed with a vagally-induced symptomatic bradydysrhythmia. Which medication does the nurse anticipate will be ordered for this patient? Digoxin Atropine Adenosine Magnesium sulfate The nurse is caring for a patient diagnosed with supraventricular tachycardia associated with excessive beta-adrenergic stimulation. Which class of antidysrhythmic drugs does the nurse anticipate being ordered for this patient? Class I Class II (LOL) Class III Class IV pp. 677, 662 The nurse is caring for a patient 2 hours postoperatively. The patient appears confused and has the following vital signs: blood pressure 88/40, pulse 125, oxygen saturation 94%, and respiratory rate 24. The nurse analyzes the cardiac rhythm and determines the patient is in sinus tachycardia. What action should the nurse take first? Call for assistance Administer digoxin IV Decrease the IV fluids Initiate the Valsalva maneuver p. 673 The nurse is caring for a patient who just had a 12-lead electrocardiogram (ECG). The nurse notices that the ECG is different from the previous ECG reading in most leads. Which action should the nurse take first? Contact the health care provider Perform a full head-to-toe physical Check the electrode placement on the patient Ask the unlicensed assistive personnel to repeat the ECG p. 684 The nurse is evaluating a patient's electrocardiogram (ECG) rhythm strip. The nurse notes the presence of a U wave. What action by the nurse is priority? Check the patient's lead placement Prepare to administer magnesium sulfate Document the findings in the patient's chart Request that a stat potassium level is obtained Rationale U waves suggest hypokalemia. If a U wave is noted, it is important to contact the health care provider to request that a stat potassium level is obtained. Lead placement does not need to be checked. Magnesium is not indicated or prescribed in this situation. Documenting the findings should be done after intervention. p. 653 The emergency room nurse is caring for a patient with a suspected myocardial infarction. What component of the patient's cardiac rhythm should be evaluated further for this suspected diagnosis? Select all that apply. P wave T wave PR interval ST segment QRS complex The nurse is caring for a patient who was admitted with chest pain. While performing a cardiac assessment, the nurse determines that the patient's apical pulse is 90, and the radial pulse is 75. What action by the nurse is priority? Contact the health care provider Document the findings in the chart Assess the patient's blood pressure Administer PRN nitroglycerin sublingual Test-Taking Tip: Once you have decided on an answer, look at the stem again. Does your choice answer the question that was asked? If the question stem asks "why," be sure the response you have chosen is a reason. If the question stem is singular, then be sure the option is singular, and the same for plural stems and plural responses. Many times, checking to make sure that the choice makes sense in relation to the stem will reveal the correct answer. p. 657 The nurse is helping to care for a patient with a heart rate of 34 bpm who is not responding to atropine. The health care provider wants to prepare for transvenous pacing. What supplies should the nurse prepare for this procedure? Select all that apply. Adenosine Pacing pads Pacing wires A central venous catheter A battery-operated pulse generator p. 674 The nurse is caring for a patient newly diagnosed with atrial fibrillation. The patient is being admitted for ibutilide fumarate treatment. For what findings should the nurse contact the health care provider? Select all that apply. Potassium of 3.7 Prolonged QT interval Previous history of atrial fibrillation Atrial fibrillation lasting for two weeks Patient report of taking anticoagulants for five days p. 678 The nurse is administering adenosine to a patient with supraventricular tachycardia. While administering the drug, the patient has a short period of asystole and then returns to normal sinus rhythm. What action by the nurse is priority? < Continue to monitor the patient Contact the health care professional Prepare to administer atropine sulfate Place defibrillation pads on the patient's chest p. 677 A patient admitted with a new diagnosis of hypertension demonstrates sinus bradycardia with a heart rate of 52 bpm on the electrocardiogram (ECG) monitor. The nurse assesses the patient and determines if the patient is asymptomatic. What actions by the nurse are priorities? Select all that apply. Increase the patient's IV fluids Administer the prescribed digoxin Document the assessment findings Contact the primary health care provider Assess the medication administration record p. 674 The nurse is caring for a patient diagnosed with bradycardia. The patient's pulse rate is between 50 to 55 bpm. The patient is hypotensive and complains of dizziness and weakness. The cause of the bradycardia is undetermined. Which treatment would be given to this patient? Select all that apply. Oxygen IV fluids Adenosine 6 mg IV push Atropine 0.5 mg IV bolus Synchronized cardioversion p. 674 A patient is transported to the emergency department because of supraventricular tachycardia. The patient is confused, and the peripheral pulses are weak. The vital signs are as follows: blood pressure of 78/35, pulse of 250, and no registration of oxygen saturation. Which action by the nurse is priority? Prepare for synchronized cardioversion Prepare diltiazem hydrochloride 0.25 mg/kg Ensure the OR is ready for immediate ablation Draw stat blood including arterial blood gas (ABG) p. 678 The nurse is caring for a patient who is undergoing synchronized cardioversion. Which interventions are the nurse responsible for after the cardioversion? Select all that apply. Maintaining a patent airway Monitoring for dysrhythmias Assessing the need for further cardioversion Assessing the chest for burns from electrodes Ordering a post-cardioversion transesophageal echocardiogram p. 681 The nurse observes this rhythm on the monitor of a patient who is unresponsive. What action should the nurse take first? Begin CPR Maintain patent airway Give two rescue breaths Check carotid pulse for 5 seconds p. 686 The patient has two premature ventricular contractions (PVCs) that are repeating in their cardiac rhythm every 3 seconds. The nurse correctly documents this as which type of rhythm? Unifocal PVC Multifocal PVC Bigeminal PVCs Nonsustained ventricular tachycardia p. 671 The nurse is performing chest compressions on a patient in asystole. Another nurse enters the room with the defibrillator. What action by the first nurse is priority? Continue compressions Clear the area of movable items Analyze the patient's cardiac rhythm Step away to allow the defibrillator to be set up p. 686 The nurse is caring for an older adult patient who is on continuous cardiac monitoring. The patient is watching the monitor and expresses concern about a wide area that has occurred twice in the last 10 minutes. The nurse recognizes that this is a premature ventricular contraction (PVC). Which statement by the nurse is appropriate? "It is OK, you are probably just moving around." "PVCs are common and increase as we get older." "I will have your health care provider come in and talk to you about this." "You may have underlying chronic obstructive pulmonary disease (COPD)." p. 683 The nurse is caring for a patient who requires a 12-lead electrocardiogram (ECG). Which interventions should the nurse perform to ensure the best signal transmission? Select all that apply. Clean and dry skin Avoid areas with scar tissue Clip hair in the area being used Use non-scented lotions on chest Ensure gel on electrodes is not moist Test-Taking Tip: Be alert for details about what you are being asked to do. In this Question Type, you are asked to select all options that apply to a given situation or patient. All options likely relate to the situation, but only some of the options may relate directly to the situation. p. 684 The nurse administers lidocaine to a patient experiencing trigeminal premature ventricular contractions (PVCs). After providing this treatment, what actions are priority? Select all that apply. Insert artificial airway Observe patient for seizures Monitor for hypomagnesemia Assess for numbness or tingling Assess for swelling of the extremities Test-Taking Tip: Identify option components as correct or incorrect. This may help you identify a wrong answer. p. 677 The nurse is providing discharge instructions to a patient diagnosed with vasovagal bradycardia. Which instruction should the nurse include? "It is important to avoid constipation." "If you feel your heart slowing, raise your hands over your head." "If you feel lightheaded or faint, it is important to try to bear down." "Gentle pressure over your carotid artery may help with increasing your heart rate." p. 682 The nurse is caring for a patient with a ventricular fibrillation. The patient has had three cycles of CPR after receiving an initial shock. When should the nurse plan to administer another shock? In approximately 4 minutes After two more cycles of CPR Defibrillation cannot be used twice After the adenosine has been administered p. 686 Fill in the blankUsing the 6-second strip method, determine the heart rate for the patient. Record your answer using a whole number. Bpm 80bpm Test-Taking Tip: Be aware that information from previously asked questions may help you respond to other questions. p. 670 Which cardiac dysrhythmia results from a prolonged QT interval? Torsades de pointe Ventricular standstill Ventricular fibrillation Premature ventricular contractions p. 670 The nurse is documenting an admission assessment of a patient scheduled for an elective cardioversion. Which statement by the patient should be reported to the health care provider immediately? < "I am really worried about feeling the shock." "I have not taken my digoxin since yesterday." "The last time I ate was a small spinach salad last night." "I have been taking my anticoagulant for the past 5 weeks." p. 684 What drug is an antidysrhythmic drug used to slow ventricular conduction? Diltiazem Apixaban Atropine sulfate Magnesium sulfate p. 678 Which action is occurring in a patient's conduction system during the circled part of the electrocardiogram (ECG) waveform? Atrial kick Atrial depolarization Ventricular contraction T-cell (transitional cell) impulse slowing Test-Taking Tip: If you can eliminate any responses as incorrect based your knowledge, you will not be guessing randomly but will be exercising "informed guessing" p. 676 Which medication is prescribed to a patient who is at high risk of emboli? Digoxin Diltiazem Metoprolol Enoxaparin p. 679 Which statement accurately describes the area of the heart that initiates the cardiac cycle? The ventricles' Purkinje fibers are responsible for the initial cardiac impulse. The bundle of His acts to initiate cardiac contraction in the atria with a resulting QRS complex. The atrioventricular area acts as the primary pacemaker and produces the P wave on the ECG. The sinoatrial (SA) node is the primary pacemaker of the heart whose activity is reflected in the P wave on the ECG. STUDY TIP: Focus your study time on the common health problems that nurses most frequently encounter. p. 665 A patient is brought into the emergency room with chest pain, shortness of breath, diaphoresis, and hypotension. The heart rate of the patient is 56 beats per minute. What is the immediate nursing intervention provided to stabilize the patient? Administering digoxin Administering atropine Administering verapamil Administering propranolol Test-Taking Tip: Start by reading each of the answer options carefully. Usually at least one of them will be clearly wrong. Eliminate this one from consideration. Now you have reduced the number of response choices by one and improved the odds. p. 674 Which dysrhythmia is caused by atrial irritability? Atrial fibrillation Premature atrial complexes Supraventricular tachycardia Premature ventricular complexes p. 676 Which complications are associated with carotid sinus massage therapy? Select all that apply. Bradycardia Cerebral damage Ventricular fibrillation Premature atrial complexes Premature ventricular complexes p. 678 A patient is prescribed adenosine drug therapy to convert supraventricular tachycardia to normal sinus rhythm. Which parameter should the nurse monitor in this patient? Bradycardia Rebound tachycardia Ventricular fibrillation Premature atrial complexes p. 678 The nurse is caring for a patient with sinus tachycardia. For which underlying causes does the nurse assess? Select all that apply. Fever Anxiety Graves' disease Straining when moving the bowels Use of a beta-adrenergic blocking drug Which cardiac rhythm typically deteriorates into ventricular fibrillation? Atrial flutter Atrial fibrillation Ventricular tachycardia Third-degree heart block p. 684 Which drugs does the nurse anticipate may be used in the treatment of a patient with atrial fibrillation? Select all that apply. Heparin Digoxin Warfarin Diltiazem Amiodarone Phytonadione (vitamin K) Test-Taking Tip: Do not read information into questions and avoid speculating. Reading into questions creates errors in judgment. p. 680 When caring for a patient with heart disease, which action may lead to serious bradydysrhythmias? Oropharyngeal suctioning Insertion of a Foley catheter Hip flexion greater than 90 degrees Walking on a treadmill at a moderate pace p. 674 How does the nurse recognize that atropine has produced a positive outcome for the patient with bradycardia? The nurse notes dyspnea. The patient states he is dizzy and weak. The patient has a heart rate of 42 beats/min. The monitor shows an increase in heart rate. p. 678 Which information does the nurse include in the discharge teaching for a patient with a newly placed implantable cardioverter/defibrillator? Select all that apply. Know how to perform cough cardiopulmonary resuscitation as instructed Go to the emergency department if the defibrillator activates three successive times When using a cell phone, hold at least 6 inches from the generator using the opposite ear Maintain Steri-Strips over the left anterior costal margin and upper abdominal area incision Remove the electrodes prior to showering or bathing and replace immediately afterwards Avoid electromagnetic interference such as leaning directly over the alternator of a running car or boat p. 688 The nurse is teaching a patient with a dysrhythmia to eat potassium-containing foods. Which food does the nurse suggest the patient include in the diet? Grapes Apples Turnips Strawberries Rationale Potassium-containing foods include citrus, tomatoes, beans, prunes, avocados, bananas, strawberries, and lettuce. Turnips, grapes, and apples are not foods high in potassium. p. 682 When teaching patients at risk for bradydysrhythmias, what information does the nurse include? < "Use a stool softener." "Stop smoking and avoid caffeine." "Avoid potassium-containing foods." "Take nitroglycerin for a slow heartbeat." p. 674 A patient has episodes of ventricular tachycardia. Which medication does the nurse anticipate administering? Digoxin Diltiazem Metoprolol Amiodarone Test-Taking Tip: Avoid selecting answers that state hospital rules or regulations as a reason or rationale for action. pp. 677, 684 Which risk factors are known to contribute to atrial fibrillation? Select all that apply. Palpitations Advancing age High blood pressure Excessive alcohol use Use of beta-adrenergic blockers p. 679 The nurse is caring for a patient with a heart rate of 143 beats/min. Which manifestations does the nurse assess the patient for? Select all that apply. Palpitations Hypoventilation Syncope Chest discomfort Dryness of the skin Test-Taking Tip: Make educated guesses when necessary. p. 672 The nurse is caring for a patient with acute coronary syndrome (ACS) and atrial fibrillation who has a new prescription for metoprolol. Which monitoring is essential when administering the medication? Troponin Heart rate Myoglobin ST segment p. 680 Which patient is appropriate for the cardiac care unit charge nurse to assign to the float RN from the medical-surgical unit? The 64-year-old patient admitted for weakness who has a sinus bradycardia with a heart rate of 58 beats/min The 71-year-old patient admitted for heart failure who is short of breath and has a heart rate of 120-130 beats/min The 88-year-old patient admitted with an elevated troponin level who is hypotensive with a heart rate of 96 beats/min The 92-year-old patient admitted with chest pain who has premature ventricular complexes and a heart rate of 102 beats/min p. 674 The nurse is assisting a nurse practitioner with the wellness examination of a high school athlete and notes the cardiac rhythm strip has all the characteristics of normal sinus rhythm, except it is irregular. There is a pattern to the irregularity associated with the patient's breathing. How does the nurse respond to this finding? Nothing, as this is a healthy rhythm Immediately notify the nurse practitioner Move the electrodes and reevaluate the rhythm strip Instruct the patient to follow up with his or her primary health care provider p. 671 A patient is taking metoprolol for supraventricular tachycardia. Which statement by the patient warrants further investigation by the nurse? "I feel constipated since I've been in the hospital." "I got dizzy when walking to the bathroom earlier." "I have been more tired since I started taking this medicine." "My pulse seems to be slower and more regular than before." Test-Taking Tip: The following are crucial requisites for doing well on the NCLEX exam: 1) A sound understanding of the subject; 2) The ability to follow explicitly the directions given at the beginning of the test; 3) The ability to comprehend what is read; 4) The patience to read each question and set of options carefully before deciding how to answer the question; 5) The ability to use the computer correctly to record answers; 6) The determination to do well; 7) A degree of confidence. p. 680 Which side effect is associated with the drug acebutolol hydrochloride? <p>Which side effect is associated with the drug acebutolol hydrochloride?</p> Tremors Insomnia Blurred vision Bronchospasm p. 677 The registered nurse teaches a student nurse about the post-discharge self-care management of a patient hospitalized with tachycardia. Which instruction given by the student nurse shows ineffective learning? "Avoid alcohol intake." "Develop strategies for stress management." "Consult a mental health professional if you have anxiety." "Include foods rich in caffeine that stimulate the central nervous system (CNS)." Test-Taking Tip: The most reliable way to ensure that you select the correct response to a multiple-choice question is to recall it. Depend on your learning and memory to furnish the answer to the question. Which condition is a result of a hypereffective heart in a well-conditioned athlete? Tachypnea Arrhythmia Bradycardia Cardiac arrest p. 674 While the health care provider is at the bedside, the patient's heart rate increases from 82 to 176 beats/min and supraventricular tachycardia is noted on the cardiac monitor. The provider decides to do immediate carotid sinus massage. Which action would the nurse take in this situation? Obtain a stat chemistry profile prior to the procedure. Remove the pillow and place a rolled towel behind the neck. Bring a defibrillator and resuscitative equipment to the bedside. Raise the side rails and place the bed in the reverse Trendelenburg position. STUDY TIP: Do not change your pattern of study. It obviously has contributed to your being here, so it worked. If you have studied alone, continue to study alone. If you have studied in a group, form a study group. p. 678 What is the greatest priority once a crash cart has been brought to a room when responding to a patient in ventricular fibrillation? Perform defibrillation Administer epinephrine Provide rescue breathing Perform cardiac compressions Test-Taking Tip: The night before the examination, you may wish to review some key concepts that you believe need additional time, but then relax and get a good night's sleep. Remember to set your alarm, allowing yourself plenty of time to dress comfortably (preferably in layers, depending on the weather), have a good breakfast, and arrive at the testing site at least 15 to 30 minutes early. p. 686 In caring for a patient following an acute myocardial infarction, what is the greatest priority when planning care? Provide a low-fat, low-sodium diet Pad the oxygen tubing behind the ears Wear TED hose and sequential compression devices while in bed Monitor for increased frequency of premature ventricular contractions A patient is admitted to the intensive care unit with an acute pulmonary embolism. What dysrhythmia would most likely contribute to this condition? Atrial fibrillation Sinus bradycardia Ventricular tachycardia Premature atrial contractions The nurse is placing a telemetry monitor on a patient being admitted for surgery who has a history of coronary artery disease. The monitor shows sinus tachycardia with a rate of 144 beats/min. How does the nurse respond to this finding? Select all that apply. Assess for jugular venous distention Assess blood pressure and skin temperature Ask the patient to report any muscle cramps Instruct the patient to ambulate as tolerated and continue to monitor Ask the patient if he or she is experiencing any chest pain or pressure Initiate high-quality CPR Determine the respiratory rate Notify the health care provider Administer 100% oxygen by nonrebreather mask The nurse is teaching a patient about the synchronous demand pacemaker that has just been implanted for an episode of complete heart block. Which statement indicates the patient understands the teaching? "When my heart beats fast, the pacemaker will correct it." "When my pulse is less than the set rate, the pacemaker stimulates my heart to beat." "If I develop a life-threatening heart rhythm, the pacemaker will give me a shock to stop it." "My heart doesn't beat right, so the pacemaker will stimulate my heart to beat 60 times per minute." Test-Taking Tip: Try putting questions and answers in your own words to test your understanding. p. 675 The nurse is assisting with resuscitation efforts on a patient in the ICU when the family tearfully comes to the door and requests to be in the room with their loved one. Which response by the nurse is best? "It is hospital policy that family do not get in the way of resuscitation attempts." "You must wait in the family waiting area. We will call you when we are finished." "I know your loved one would not want you to see him like this. I'll put a chair right outside the room." "If you feel you need to be present for your loved one, remain to the side and I will answer your questions later." When caring for a patient who has been admitted with tachydysrhythmia, which assessment finding would the nurse expect? Fainting Flushing Agitation Bounding pulse p. 672 A patient with atrial fibrillation with rapid ventricular response has received medication to slow the ventricular rate. The pulse is now 88 beats/min. For which additional therapy does the nurse plan? <p>A patient with atrial fibrillation with rapid ventricular response has received medication to slow the ventricular rate. The pulse is now 88 beats/min. For which additional therapy does the nurse plan?</p> Anticoagulation Synchronized cardioversion Electrophysiology studies (EPS) Radiofrequency ablation therapy p. 680 A 78-year-old patient has recently been prescribed a daily dose of diltiazem for atrial fibrillation. Her daughter calls to report that she is experiencing weakness and confusion for the past couple of days. How does the nurse respond to the daughter's concern? <p>A 78-year-old patient has recently been prescribed a daily dose of diltiazem for atrial fibrillation. Her daughter calls to report that she is experiencing weakness and confusion for the past couple of days. How does the nurse respond to the daughter's concern?</p> "Tell your mother to cut the dose in half." "Tell your mother to take the dose every other day." "Has your mother experienced any dizziness during this time?" "Your mother is getting older and confusion is common at her age." Test-Taking Tip: Do not select answers that contain exceptions to the general rule, controversial material, or degrading responses. p. 678 The bedside cardiac monitor alarms, and the patient suddenly becomes limp and unresponsive with no carotid pulse. What should the nurse do first? Maintain a patent airway Start cardiac compressions Call for the Rapid Response Team Place a firm board under the patient The nurse is the second responder for a person who collapsed in a school and does not have a pulse. After retrieving the automated external defibrillator (AED), the nurse applies the pads, plugs them in, and starts the machine. The AED indicates the pulseless patient is in ventricular asystole. What is the nurse's priority action? Start compressions Initiate an airway in the patient Ensure everyone is "clear" for defibrillation Place electrodes on the chest and initiate transcutaneous pacing Test-Taking Tip: Read every word of each question and option before responding to the item. Glossing over the questions just to get through the examination quickly can cause you to misread or misinterpret the real intent of the question. p. 686 The nurse is caring for a patient who has developed bradycardia. Which possible causes does the nurse investigate? Select all that apply. Diltiazem administered 1 hour ago Bearing down for a bowel movement Patient stating that he just had a cup of coffee Possible inferior wall myocardial infarction (MI) Patient becoming emotional when visitors arrived . STUDY TIP: Do not change your pattern of study. It obviously has contributed to your being here, so it worked. If you have studied alone, continue to study alone. If you have studied in a group, form a study group. p. 674 The nurse is caring for a patient with advanced heart failure who develops asystole. The nurse corrects the graduate nurse when the graduate offers to perform which intervention? Defibrillation Administration of oxygen Administration of epinephrine Cardiopulmonary resuscitation (CPR) p. 686 Which precaution should the nurse follow when providing cardiopulmonary resuscitation (CPR) and advanced cardiac life support (ACLS) to a patient? Protective Isolation Standard Precautions Surgical asepsis with defibrillator Respiratory isolation during intubation p. 686 The nurse is teaching a patient with a new permanent pacemaker. Which statement by the patient indicates a need for further discharge education? <p>The nurse is teaching a patient with a new permanent pacemaker. Which statement by the patient indicates a need for further discharge education?</p> "I no longer need my heart pills." "I need to take my pulse every day." "I will be able to shower again soon." "I might trigger airport security metal detectors." p. 688 What teaching does the nurse include for a patient with atrial fibrillation who has a new prescription for warfarin? <p>What teaching does the nurse include for a patient with atrial fibrillation who has a new prescription for warfarin?</p> "Avoid caffeinated beverages." "Report nosebleeds to your provider immediately." "You should take aspirin or ibuprofen for headache." "It is important to consume a diet high in green leafy vegetables." p. 680 The nurse receives a report that a patient with a pacemaker has experienced loss of capture. Which situation is consistent with this? <p>The nurse receives a report that a patient with a pacemaker has experienced loss of capture. Which situation is consistent with this?</p> The patient demonstrates hiccups The pacemaker spike falls on the T wave Pacemaker spikes are noted, but no P wave or QRS complex follows The heart rate is 42 beats/min, and no pacemaker spikes are seen on the rhythm strip p. 675 Which teaching is essential for a patient who has had a permanent pacemaker inserted? <p>Which teaching is <b>essential</b> for a patient who has had a permanent pacemaker inserted?</p> Avoid sexual activity Do not take tub baths Avoid talking on a cell phone Avoid operating electrical appliances over the pacemaker p. 688 Which statement correctly differentiates cardioversion from defibrillation? <p>Which statement correctly differentiates cardioversion from defibrillation?</p> Defibrillation is a synchronized shock delivered to depolarize the myocardium simultaneously in atrial fibrillation. Cardioversion is an asynchronous shock to the patient to convert ventricular tachycardia or ventricular fibrillation. Defibrillation delivers an electrical shock to the heart; cardioversion involves use of a temporary pacemaker to deliver the shock. Cardioversion delivers a synchronized shock for ventricular tachycardia or supraventricular tachycardia. p. 681 The nurse is teaching a patient who has been diagnosed with atrial fibrillation. Which food item does the nurse teach the patient to avoid? <p>The nurse is teaching a patient who has been diagnosed with atrial fibrillation. Which food item does the nurse teach the patient to avoid?</p> Sodium Caffeine Citrus products Low-fiber foods p. 682 The nurse is caring for a patient with a tachydysrhythmia who asks why his chest hurts. The nurse should base the explanation on which information? <p>The nurse is caring for a patient with a tachydysrhythmia who asks why his chest hurts. The nurse should base the explanation on which information?</p> Heart rate over 100 beats per minute may cause palpitations. The diastole is shortened, and coronary perfusion is decreased. The aortic valve remains open, preventing coronary artery filling. The conduction is delayed at the atrioventricular (AV) node, causing causing heart block. Test-Taking Tip: Make certain that the answer you select is reasonable and obtainable under ordinary circumstances and that the action can be carried out in the given situation. p. 672 The nurse is receiving shift change report on a patient with an implantable cardioverter/defibrillator (ICD). Which test or activity, if ordered by the provider, would prompt the nurse to contact the provider immediately? <p>The nurse is receiving shift change report on a patient with an implantable cardioverter/defibrillator (ICD). Which test or activity, if ordered by the provider, would prompt the nurse to contact the provider immediately?</p> Radiation therapy Radiographic imaging Computed tomography Magnetic resonance imaging p. 675 The patient has had asymptomatic sinus bradycardia with a rate of 56 beats/min. The nurse notes the rate has dropped to 46 beats/min. What action does the nurse take? <p>The patient has had asymptomatic sinus bradycardia with a rate of 56 beats/min. The nurse notes the rate has dropped to 46 beats/min. What action does the nurse take?</p> Increase IV fluid flow rate Notify the Rapid Response Team Administer atropine 0.5 mg IV push Assess blood pressure, skin color, and moisture Test-Taking Tip: Do not select answers that contain exceptions to the general rule, controversial material, or degrading responses. p. 674 To prevent vagal stimulation in a patient with heart disease, what must be avoided? Select all that apply. <p>To prevent vagal stimulation in a patient with heart disease, what must be avoided? <b>Select all that apply.</b> </p> Hiccups Constipation Oropharyngeal suctioning Insertion of a nasogastric tube Hip flexion greater than 90 degrees p. 674 When caring for a patient with premature ventricular contractions (PVCs), which electrolyte imbalances will contribute to this dysrhythmia and should therefore be monitored? Select all that apply. <p>When caring for a patient with premature ventricular contractions (PVCs), which electrolyte imbalances will contribute to this dysrhythmia and should therefore be monitored? <b>Select all that apply.</b> </p> Hypokalemia Hyponatremia Hypocalcemia Hypomagnesemia Hypophosphatemia pp. 672, 684 A college student presents to the campus clinic reporting palpitations. Frequent premature atrial contractions are identified on the electrocardiogram (ECG). What does the nurse include in teaching this patient? Select all that apply. <p>A college student presents to the campus clinic reporting palpitations. Frequent premature atrial contractions are identified on the electrocardiogram (ECG). What does the nurse include in teaching this patient? <b>Select all that apply.</b> </p> Smoking cessation Limited physical exertion Decreased consumption of caffeinated beverages Improvement of hydration with increased water intake Referral to the counseling center for stress management p. 676 The nurse is caring for a patient on a telemetry unit with a regular heart rhythm and rate of 60 beats/min; a P wave precedes each QRS complex, and the PR interval is 0.24 second. Additional vital signs are as follows: blood pressure 118/68, respiratory rate 16, and temperature 98.8° F. All of these medications are available on the medication record. What action does the nurse take? <p>The nurse is caring for a patient on a telemetry unit with a regular heart rhythm and rate of 60 beats/min; a P wave precedes each QRS complex, and the PR interval is 0.24 second. Additional vital signs are as follows: blood pressure 118/68, respiratory rate 16, and temperature 98.8&#xb0; F. All of these medications are available on the medication record. What action does the nurse take?</p> Administer digoxin Administer atropine Continue to monitor Administer clonidine p. 671 The nurse is caring for a patient with atrial fibrillation. In addition to an antidysrhythmic, what medication does the nurse plan to administer? <p>The nurse is caring for a patient with atrial fibrillation. In addition to an antidysrhythmic, what medication does the nurse plan to administer?</p> Rivaroxaban Atropine Dobutamine Magnesium sulfate p. 684 A patient admitted after using crack cocaine develops ventricular fibrillation. After determining unresponsiveness, which action does the nurse take next? <p>A patient admitted after using crack cocaine develops ventricular fibrillation. After determining unresponsiveness, which action does the nurse take <b>next</b>?</p> Establish IV access Prepare for defibrillation Place an oral airway and ventilate Start cardiopulmonary resuscitation (CPR) Test-Taking Tip: A psychological technique used to boost your test-taking confidence is to look into a mirror whenever you pass one and say out loud, "I know the material, and I'll do well on the test." Try it; many students have found that it works because it reduces "test anxiety." p. 686 The nurse is caring for a patient with unstable angina whose cardiac monitor shows ventricular tachycardia. Which action is appropriate to implement first? <p>The nurse is caring for a patient with unstable angina whose cardiac monitor shows ventricular tachycardia. Which action is appropriate to implement <b>first</b>?</p> Check the patient for a pulse Cardiovert the patient at 50 J Give the patient IV lidocaine Defibrillate the patient at 200 J p. 684 Which complex makes up a normal electrocardiogram (ECG)? <p>Which complex makes up a normal electrocardiogram (ECG)?</p> QRS PQR RST STU p. 667 The registered nurse teaches a student nurse about continuous electrocardiogram (ECG) monitoring and the skin and electrode preparations that could affect the ECG recordings. Which statement by the student nurse indicates the need for further teaching? <p>The registered nurse teaches a student nurse about continuous electrocardiogram (ECG) monitoring and the skin and electrode preparations that could affect the ECG recordings. Which statement by the student nurse indicates the need for further teaching?</p> "The area for electrode placement must be moist." "Electrodes should not be placed on irritated skin or over scar tissue." "The skin should be cleaned and the hair should be clipped if needed." "The electrode is attached to the lead cable and then to the contact site." p. 667 The nurse is reviewing the electrocardiogram (ECG) of a patient with cardiac dysrhythmias and observes shape changes in the P wave in the given lead. What is the possible reason for these changes in the P wave? <p>The nurse is reviewing the electrocardiogram (ECG) of a patient with cardiac dysrhythmias and observes shape changes in the P wave in the given lead. What is the possible reason for these changes in the P wave?</p> Ectopic focus Myocardial ischemia Electrolyte imbalances Coronary artery disease Test-Taking Tip: Identify option components as correct or incorrect. This may help you identify a wrong answer. p. 667 What is the true isoelectric interval in the electrocardiogram (ECG)? <p>What is the true isoelectric interval in the electrocardiogram (ECG)?</p> PR segment ST segment TP segment TU segment p. 667 A female patient who is on amiodarone medication complains of chest pain, dizziness, and heart palpitations. The electrocardiogram (ECG) of the patient shows QT interval abnormalities. What condition do these findings indicate? <p>A female patient who is on amiodarone medication complains of chest pain, dizziness, and heart palpitations. The electrocardiogram (ECG) of the patient shows QT interval abnormalities. What condition do these findings indicate?</p> Torsades de pointes (TDP) Congestive heart failure (CHF) Hypertrophic cardiomyopathy (HCM) Paroxysmal supraventricular tachycardia (PSVT) Test-Taking Tip: Read the question carefully ; make certain that the answer you select is reasonable and obtainable under ordinary circumstances. p. 670 The registered nurse is teaching a student nurse about the causes of artifacts on an electrocardiogram (ECG). Which statement by the student nurse needs correction? <p>The registered nurse is teaching a student nurse about the causes of artifacts on an electrocardiogram (ECG). Which statement by the student nurse needs correction?</p> "Using loose electrodes during recording causes artifacts." "Grounding the equipment during recording causes artifacts." "Unnecessary patient movements during recording cause artifacts." "Faulty electrodes and equipment used during recording cause artifacts." Test-Taking Tip: Sometimes the reading of a question in the middle or toward the end of an exam may trigger your mind with the answer or provide an important clue to an earlier question. p.670 Which parameter is determined by analyzing the P waves on an electrocardiogram (ECG) using the walking out technique? <p>Which parameter is determined by analyzing the P waves on an electrocardiogram (ECG) using the walking out technique?</p> Tachycardia Bradycardia Atrial rhythm Intensity of atrial depolarization p. 670 The nurse observes abnormal T waves in the electrocardiogram of a patient with dysrhythmia. What is the possible reason for the patient's dysrhythmia? <p>The nurse observes abnormal T waves in the electrocardiogram of a patient with dysrhythmia. What is the possible reason for the patient's dysrhythmia?</p> Ectopic focus Myocardial infarction Electrolyte imbalances Coronary artery disease Test-Taking Tip: Identifying content and what is being asked about that content is critical to your choosing the correct response. Be alert for words in the stem of the item that are the same or similar in nature to those in one or two of the options. p. 670 Which equipment is involved in monitoring of cardiac electrical activity during electrocardiography? <p>Which equipment is involved in monitoring of cardiac electrical activity during electrocardiography?</p> A computer held by the patient A clicker system held close to the chest Multiple electrodes attached to the body A global positioning system (GPS) transmitting the signal Test-Taking Tip: Choose the best answer for questions asking for a single answer. More than one answer may be correct, but one answer may contain more information or more important information than another. p. 665 A patient is having a 12-lead electrocardiogram (ECG) to rule out a heart dysrhythmia. The patient shows a positive deflection on the ECG strip. How does the nurse interpret this result? <p>A patient is having a 12-lead electrocardiogram (ECG) to rule out a heart dysrhythmia. The patient shows a positive deflection on the ECG strip. How does the nurse interpret this result?</p> The cardiac axis is moving neither toward nor away from the axis. There is no current flow after complete depolarization and repolarization. The electrical current flow in the heart (cardiac axis) is toward the positive pole. The electrical current flow in the heart (cardiac axis) is toward the negative pole. An adult patient is scheduled for a 12-lead electrocardiogram (ECG) to rule out a dysrhythmia. Four of the leads are placed on the extremities. Which extremity is used for the ground electrode? <p>An adult patient is scheduled for a 12-lead electrocardiogram (ECG) to rule out a dysrhythmia. Four of the leads are placed on the extremities. Which extremity is used for the ground electrode?</p> Left leg Left arm Right leg Right arm Test-Taking Tip: Multiple-choice questions can be challenging, because students think that they will recognize the right answer when they see it or that the right answer will somehow stand out from the other choices. This is a dangerous misconception. The more carefully the question is constructed, the more each of the choices will seem like the correct response. p. 666 A 12-lead electrocardiogram (ECG) is being performed on a female patient for the purpose of cardiac evaluation. What should the nurse include in the teaching when instructing the patient about the ECG test? Select all that apply. <p>A 12-lead electrocardiogram (ECG) is being performed on a female patient for the purpose of cardiac evaluation. What should the nurse include in the teaching when instructing the patient about the ECG test? <b>Select all that apply.</b> </p> "Remain as still as possible." "You will not be permitted to eat." "Breathe normally without repetitive movement." "Your bed will be moved into the Trendlenburg position." "You may need to move your breasts so leads can be placed on your chest." When placing electrodes on the patient for continuous electrocardiographic monitoring, the nurse places the electrodes on which locations? Select all that apply. <p>When placing electrodes on the patient for continuous electrocardiographic monitoring, the nurse places the electrodes on which locations? <b>Select all that apply.</b> </p> Below the scapula Near the metatarsal Below the left clavicle Below the right clavicle On the lowest palpable rib, left mid-clavicular line On the lowest palpable rib, right mid-clavicular line Test-Taking Tip: All options likely relate to the situation but only some of the options relate directly to the situation. pp. 666-667 A patient has an abnormal PR segment on the electrocardiogram (ECG). What part of the heart does the nurse attribute to the abnormal wave? <p>A patient has an abnormal PR segment on the electrocardiogram (ECG). What part of the heart does the nurse attribute to the abnormal wave?</p> SA node AV node Left and right ventricles The right bundle branch Rationale The AV node is where the impulses of the transitional cells of the right atria slow down or are delayed and would be reflected by the PR segment on the ECG. The P wave reflects the SA node. Impulses in the left and right ventricles and the right bundle branch are reflected by the QRS complex. Test-Taking Tip: The most reliable way to ensure that you select the correct response to a multiple-choice question is to recall it. p. 669 Many acute care hospitals have monitor technicians on their telemetry floors to follow patients being monitored by continuous electrocardiogram (ECG). What are the responsibilities of the monitor technician? Select all that apply. <p>Many acute care hospitals have monitor technicians on their telemetry floors to follow patients being monitored by continuous electrocardiogram (ECG). What are the responsibilities of the monitor technician? <b>Select all that apply.</b> </p> Interpret rhythms and report Assess monitored patients every 4 hours Print ECG rhythm strips routinely and as needed Have ongoing communication with the physicians Continually watch the bank of monitors on the acute care unit Report the patient's rhythm and significant changes to the nurse p. 667 The nurse evaluates the placement of the electrodes to ensure clarity of monitoring. What ensures appropriate electrode preparation and placement? Select all that apply. <p>The nurse evaluates the placement of the electrodes to ensure clarity of monitoring. What ensures appropriate electrode preparation and placement? <b>Select all that apply.</b> </p> The electrode gel is moist The doctor placed the electrodes Electrodes were placed on scar tissue No lotion is present on the patient's skin The nurse selected the leads and checked placement Chest hair is shaved or clipped where electrodes will be placed p. 667 What is important for the nurse to know about the P wave in sinus rhythm when examining a rhythm strip? <p>What is important for the nurse to know about the P wave in sinus rhythm when examining a rhythm strip?</p> It precedes the QRS complex. It is always positively deflected. It is followed immediately by a T wave. It is a deflection representing atrial repolarization. Test-Taking Tip: Key words or phrases in the question stem such as first, primary, early, or best are important. Similarly, words such as only, always, never, and all in the alternatives are frequently evidence of a wrong response. No real absolutes exist in life; however, every rule has its exceptions, so answer with care. p. 671 Which criteria does the normal sinus rhythm meet? Select all that apply. <p>Which criteria does the normal sinus rhythm meet? <b>Select all that apply.</b> </p> It originates from the SA node. The QRS duration is 0.04-0.10 seconds. The PR interval measures 0.10-0.20 seconds. It has atrial and ventricular rates of 70-100 beats/min. Atrial and ventricular rhythms may be regular or irregular. p. 671 The patient's telemetry reading shows a P wave before each QRS complex, and the rate is 78 beats/minute. Which action should the nurse implement? <p>The patient's telemetry reading shows a P wave before each QRS complex, and the rate is 78 beats/minute. Which action should the nurse implement?</p> Prepare to administer digoxin PO Assess the patient's cardiac enzymes Request a 12-lead electrocardiogram Document it as normal sinus rhythm (NSR) Test-Taking Tip: You have at least a 25% chance of selecting the correct response in multiple-choice items. If you are uncertain about a question, eliminate the choices that you believe are wrong and then call on your knowledge, skills, and abilities to choose from the remaining responses. p. 671 How would the nurse interpret the cardiac rhythm in the figure? <p>How would the nurse interpret the cardiac rhythm in the figure? <img src="https://eolscontent.elsevier.com/1069FQX38FT/image/1191_iggy_ch34_fig6.png" alt="1191_iggy_ch34_fig6.png"/> </p> Sinus arrhythmia Sinus tachycardia Sinus bradycardia Normal sinus rhythm p. 671 What complexes make up a normal electrocardiogram (ECG)? Select all that apply. <p>What complexes make up a normal electrocardiogram (ECG)? <b>Select all that apply.</b> </p> S wave P wave T wave U wave QRS complex p. 671 What does the PR interval represent on the electrocardiogram (ECG)? <p>What does the PR interval represent on the electrocardiogram (ECG)?</p> Atrial depolarization Ventricular depolarization Impulse traveling to the base of the ventricle Impulse traveling from the atrium to the ventricles Test-Taking Tip: Try putting questions and answers in your own words to test your understanding. p. 667 What represents no electrical current flow in the heart on the electrocardiogram (ECG)? <p>What represents no electrical current flow in the heart on the electrocardiogram (ECG)?</p> Cardiac axis Isoelectric line Positive deflections Negative deflections p. 666 Which is a characteristic of the ST segment? <p>Which is a characteristic of the ST segment?</p> It is normally a positive deflection. It represents ventricular depolarization. Its length is consistent in measurement. It occurs from the J point to the beginning of the T wave. p. 669 The nurse wants to determine the patient's average heart rate by observing the electrocardiogram (ECG) strip. She counts the number of QRS complexes documented within 6 seconds and multiplies that number by 10. What is this method called? <p>The nurse wants to determine the patient's average heart rate by observing the electrocardiogram (ECG) strip. She counts the number of QRS complexes documented within 6 seconds and multiplies that number by 10. What is this method called?</p> Memory method Big block method Commercial method The 6-second method p. 670 The nurse prints out the following 6-second strip and analyzes the rhythm using the eight-step method. How many seconds is the PR interval? <p>The nurse prints out the following 6-second strip and analyzes the rhythm using the eight-step method. How many seconds is the PR interval? <img src="https://eolscontent.elsevier.com/1069FQX38FT/image/1228_iggy_ch34_fig7.png" alt="1228_iggy_ch34_fig7.png"/> </p> 0.04 seconds 0.08 seconds 0.12 seconds 0.16 seconds p. 670 The nurse is reviewing the patient's continuous ECG and notes a depression of the ST segment. About which conditions should the nurse be concerned? Select all that apply. <p>The nurse is reviewing the patient's continuous ECG and notes a depression of the ST segment. About which conditions should the nurse be concerned? <b>Select all that apply.</b> </p> Pericarditis Hypokalemia Hyperkalemia Myocardial infarction Ventricular hypertrophy Rationale ST depression is associated with hypokalemia, myocardial infarction, or ventricular hypertrophy. ST elevation may indicate pericarditis and hyperkalemia. p. 670 The nurse is reviewing the medical history of a patient recently admitted to the floor and notes the patient has sinus arrhythmia. The nurse knows which information about sinus arrhythmia is true? Select all that apply. <p>The nurse is reviewing the medical history of a patient recently admitted to the floor and notes the patient has sinus arrhythmia. The nurse knows which information about sinus arrhythmia is true? <b>Select all that apply.</b> </p> It is an irregular rhythm. It means absence of rhythm. It has all the characteristics of normal sinus rhythm. It is frequently observed in patients with coronary heart disease. The heart rate increases and decreases slightly during respirations. Rationale Sinus arrhythmia is an irregular rhythm in which the heart rate increases slightly during inspiration and decreases slightly during exhalation. It does not mean absence of rhythm, as the term suggests. It has the characteristics of NSR except for its irregularity and is frequently observed in healthy adults, not in patients with coronary heart disease. p. 671 In addition to the 12-lead electrocardiogram (ECG), other lead systems include the 18-lead ECG. What is the advantage of the 18-lead ECG? <p>In addition to the 12-lead electrocardiogram (ECG), other lead systems include the 18-lead ECG. What is the advantage of the 18-lead ECG?</p> It detects valve regurgitation issues. It offers a better view of the four major valves. It provides more information about the left ventricle. It provides more information about the right side of the heart. p. 666 When determining irregular heart rate on an electrocardiogram (ECG) rhythm strip, which method is used for best accuracy? <p>When determining irregular heart rate on an electrocardiogram (ECG) rhythm strip, which method is used for <b>best</b> accuracy?</p> ECG rate ruler Memory method Big block method 6-second strip method p. 670 How does the nurse establish that an impulse from one focus is responsible for both atrial and ventricular depolarization? <p>How does the nurse establish that an impulse from one focus is responsible for both atrial and ventricular depolarization?</p> Analyze whether the P wave shape is consistent. Determine whether impulses originate from the SA node. Determine whether intervals are greater than 0.20 seconds. Check that one P wave occurs before each QRS complex. Test-Taking Tip: Multiple-choice questions can be challenging, because students think that they will recognize the right answer when they see it or that the right answer will somehow stand out from the other choices. This is a dangerous misconception. The more carefully the question is constructed, the more each of the choices will seem like the correct response. p. 670 The nurse is caring for a patient with hypokalemia. Which abnormality does the nurse notice on the electrocardiogram (ECG)? <p>The nurse is caring for a patient with hypokalemia. Which abnormality does the nurse notice on the electrocardiogram (ECG)?</p> U wave present Prolonged PR interval An elevated ST segment Abnormally present Q wave p. 671 A patient is recovering from a myocardial infarction but has developed a dysrhythmia. The patient asks the nurse what caused the change in rhythm. Which explanation by the nurse is accurate? <p>A patient is recovering from a myocardial infarction but has developed a dysrhythmia. The patient asks the nurse what caused the change in rhythm. Which explanation by the nurse is accurate?</p> "Negatively charged cells have gained a positive charge." "The heart rate has increased, causing a change in rhythm." "The dysrhythmia was caused by blood moving through the heart efficiently." "Other cardiac cells in addition to the SA node create electrical impulses, causing a dysrhythmia." p. 665 A patient is being continuously monitored on the telemetry unit and has had stable rhythm, as noted on the electrocardiogram (ECG) strip. The patient asks when his monitor can be discontinued. What is an appropriate response by the nurse? <p>A patient is being continuously monitored on the telemetry unit and has had stable rhythm, as noted on the electrocardiogram (ECG) strip. The patient asks when his monitor can be discontinued. What is an <b>appropriate</b> response by the nurse?</p> "Monitoring can be discontinued at bedtime." "Monitoring cannot be discontinued for any reason." "You can determine when monitoring can be discontinued." "Monitoring can usually be suspended for showering and transport to other units." p. 667 When analyzing the ST segment on the electrocardiogram (ECG) strip, the nurse notices ST depression. Additionally, the patient has a potassium level of 3.1 mEq/L. Which foods does the nurse encourage the patient to increase in his diet? <p>When analyzing the ST segment on the electrocardiogram (ECG) strip, the nurse notices ST depression. Additionally, the patient has a potassium level of 3.1 mEq/L. Which foods does the nurse encourage the patient to increase in his diet?</p> Pretzels and beef broth Blueberries and cabbage Bananas and orange juice Salted nuts and canned beans p. 682 Where does the nurse place the electrodes on the patient for continuous electrocardiogram (ECG) monitoring? <p>Where does the nurse place the electrodes on the patient for continuous electrocardiogram (ECG) monitoring?</p> On the extremities On the shoulder blades On the trunk of the body On the chest over the heart p. 667 Who is responsible for determining when continuous electrocardiogram (ECG) monitoring can be suspended, such as during showering? <p>Who is responsible for determining when continuous electrocardiogram (ECG) monitoring can be suspended, such as during showering?</p> The nurse The patient The monitor technician The health care provider p. 667 To accurately monitor the patient's heart rhythm, the nurse must be able to analyze the electrocardiogram (ECG) strip. Which statements are true regarding ECG paper? Select all that apply. <p>To accurately monitor the patient's heart rhythm, the nurse must be able to analyze the electrocardiogram (ECG) strip. Which statements are true regarding ECG paper? <b>Select all that apply.</b> <img src="https://eolscontent.elsevier.com/1069FQX38FT/image/1035_iggy_ch34_fig8.png" alt="1035_iggy_ch34_fig8.png"/> </p> Time is measured horizontally Each large box measures 0.20 seconds Each small box measures 0.04 seconds One large block includes four small boxes Each large box measures 1 mm height and 1 mm width p. 667 When analyzing a cardiac rhythm strip, which component on the waveform varies with changes in heart rate? <p>When analyzing a cardiac rhythm strip, which component on the waveform varies with changes in heart rate?</p> PR interval ST segment QT interval QRS duration p. 670 Which description is characteristic of normal sinus rhythm? <p>Which description is characteristic of normal sinus rhythm?</p> The T wave is followed by a U wave. The PP intervals are essentially the same. The PR interval is greater than 0.20 seconds. There is a P wave for every QRS complex. Test-Taking Tip: Multiple-choice questions can be challenging, because students think that they will recognize the right answer when they see it or that the right answer will somehow stand out from the other choices. This is a dangerous misconception. The more carefully the question is constructed, the more each of the choices will seem like the correct response. p. 671 The nurse is preparing to perform a 12-lead electrocardiogram (ECG) on a patient. Where does the nurse place the first V-lead (precordial lead)? Left leg toward the ankle Left arm toward the wrist Fourth intercostal space, right sternal border Fourth intercostal space, left sternal border p. 666 Which statements are true regarding artifacts seen on the monitor or rhythm strip? Select all that apply. <p>Which statements are true regarding artifacts seen on the monitor or rhythm strip? <b>Select all that apply.</b> </p> An artifact may look like a fuzzy baseline. An artifact may look like a straight, narrow line. An artifact may mimic a lethal dysrhythmia. The monitor is more reliable than patient assessment. An artifact can be caused by patient movement or loose electrodes. 670 A 12-lead electrocardiogram (ECG) is ordered for a patient in an outpatient clinic. The patient expresses some concern and nervousness about the test. What should the nurse include in the teaching to decrease the patient's potential anxiety? <p>A 12-lead electrocardiogram (ECG) is ordered for a patient in an outpatient clinic. The patient expresses some concern and nervousness about the test. What should the nurse include in the teaching to decrease the patient's potential anxiety?</p> Electrodes will be placed on the chest and legs Acetaminophen must be given an hour before the test A contrast agent will be injected, but it will not cause pain No food or liquids are permitted up to 4 hours before the test Test-Taking Tip: Key words or phrases in the question stem such as first, primary, early, or best are important. Similarly, words such as only, always, never, and all in the alternatives are frequently evidence of a wrong response. No real absolutes exist in life; however, every rule has its exceptions, so answer with care. p. 666 Contractility, or the mechanical activity of the heart, occurs in a specific order to push the blood forward. What is the order of conduction that best identifies the contractility of the heart? <p>Contractility, or the mechanical activity of the heart, occurs in a specific order to push the blood forward. What is the order of conduction that <b>best</b> identifies the contractility of the heart?</p> Bundle of His, AV node, SA node, left bundle branch AV node, bundle of His, left bundle branch, SA node SA node, AV node, bundle of His, left bundle branch Left bundle branch, AV node, SA node, bundle of His pp. 664-665 Which technique is used by the nurse to calculate the accurate heart rate for a patient who shows irregular rhythms on the electrocardiogram (ECG) strip? <p>Which technique is used by the nurse to calculate the accurate heart rate for a patient who shows irregular rhythms on the electrocardiogram (ECG) strip?</p> Memory method Big block method Rule of 300 method 6-second strip method 10. The memory method is widely used in hospitals to calculate heart rates for regular rhythms. The big block method, also called the rule of 300 method, is used when the QRS complexes are regular or evenly spaced. p. 670 Which steps are involved in the electrocardiographic rhythm analysis of P waves? Select all that apply. <p>Which steps are involved in the electrocardiographic rhythm analysis of P waves? <b>Select all that apply.</b> </p> Looking for the presence of P waves Analyzing the similarity in appearance of P waves Assessing whether P waves are greater than 0.20 seconds Assessing the consistent occurrence of one P wave for each QRS complex Placing one caliper point at the start of the P wave and the other at the end of the PR segment p. 670 In which location would the nurse place lead II for a positive deflection electrocardiogram (ECG)? <p>In which location would the nurse place lead II for a positive deflection electrocardiogram (ECG)?</p> Left arm or under the heart Left leg or lowest rib, left midclavicular line Fifth intercostal space, left midclavicular line Fourth intercostal space, left of the sternal border Test-Taking Tip: The most reliable way to ensure that you select the correct response to a multiple-choice question is to recall it. p. 666 Fill in the blankThe nurse is calculating the patient's heart rate from a 6-second rhythm strip. There are eight QRS complexes in the strip. How does the nurse document the patient's estimated heart rate? Record your answer using a whole number. beats per minute Rationale The heart rate (beats per minute) can be estimated by counting the number of QRS complexes in 6 seconds and multiplying that number by 10; it is a quick but imprecise method to determine the mean or average heart rate. 8 × 10 = 80. p. 670 In electrocardiography, what is the normal time measurement of the PR interval? <p>In electrocardiography, what is the normal time measurement of the PR interval?</p> 0.04-0.12 second 0.12-0.20 second 0.20-0.28 second 0.28-0.36 second Rationale The PR interval normally measures 0.12-0.20 second. p. 669 Which finding on a cardiac rhythm strip indicates there is a delay in conduction of the electrical impulse? <p>Which finding on a cardiac rhythm strip indicates there is a delay in conduction of the electrical impulse?</p> PR interval 0.16 second PR interval of 0.08 second QRS duration of 0.10 second QRS duration of 0.24 second Test-Taking Tip: As you answer each question, write a few words about why you think that answer is correct; in other words, justify why you selected that answer. If an answer you provide is a guess, mark the question to identify it. This will permit you to recognize areas that need further review. It will also help you to see how correct your "guessing" can be. Remember: on the licensure examination you must answer each question before moving on to the next question. p. 669 A patient's rhythm strip shows a heart rate of 116 beats/min, one P wave occurring before each QRS complex, a PR interval measuring 0.16 second, and a QRS complex measuring 0.08 second. How does the nurse interpret this rhythm strip? <p>A patient's rhythm strip shows a heart rate of 116 beats/min, one P wave occurring before each QRS complex, a PR interval measuring 0.16 second, and a QRS complex measuring 0.08 second. How does the nurse interpret this rhythm strip?</p> Sinus tachycardia Sinus bradycardia Normal sinus rhythm Sinus rhythm with premature ventricular contractions p. 673 The professional nurse is supervising a nursing student performing a 12-lead electrocardiogram (ECG). Under which circumstance does the nurse correct the student? <p>The professional nurse is supervising a nursing student performing a 12-lead electrocardiogram (ECG). Under which circumstance does the nurse correct the student?</p> The patient is instructed to lie still. The patient is semi-recumbent in bed. Chest leads are placed as for the previous ECG. The patient is instructed to breathe deeply through the mouth. p. 667 What instruction should the nurse give a family member caring for the patient who has a permanent pacemaker? <p>What instruction should the nurse give a family member caring for the patient who has a permanent pacemaker?</p> "Take the pulse for one full minute at the same time each day." "Take the pulse for two full minutes at the same time each day." "Take the pulse for one full minute at different times each day." "Take the pulse for two full minutes at different times each day." Test-Taking Tip: Avoid looking for an answer pattern or code. There may be times when four or five consecutive questions have the same letter or number for the correct answer p. 676 A nurse is evaluating the understanding of family members who are caring for an older adult who has just received a permanent pacemaker. Which family member's statement requires nursing intervention? <p>A nurse is evaluating the understanding of family members who are caring for an older adult who has just received a permanent pacemaker. Which family member's statement requires nursing intervention?</p> "I should make sure the patient wears tight clothing over the generator." "I should be sure the patient doesn't lean over any electrical or gasoline engines or motors." "I should try to keep the patient away from antitheft devices in stores as much as possible." "I should tell airport personnel about the pacemaker before the patient goes through a metal detector." p. 676 What are the causes of atrial irritability? Select all that apply. <p>What are the causes of atrial irritability? <b>Select all that apply.</b> </p> Stress Fatigue Anemia Infection Muscle atrophy p. 676 The nurse reviews a patient's electrocardiogram (ECG) and notes that there is no current flow in the heart after complete depolarization as well as after complete repolarization. What does the nurse understand about this finding? <p>The nurse reviews a patient's electrocardiogram (ECG) and notes that there is no current flow in the heart after complete depolarization as well as after complete repolarization. What does the nurse understand about this finding?</p> This is the patient's isoelectric line. The patient has a positive deflection. This is the representative negative deflection. The patient is experiencing distress in the lead axis. p. 666 The nurse supervises as a student nurse places electrodes on a patient for continuous electrocardiographic monitoring. Which action made by the student nurse causes the nurse to intervene? <p>The nurse supervises as a student nurse places electrodes on a patient for continuous electrocardiographic monitoring. Which action made by the student nurse causes the nurse to intervene?</p> Placing the left leg electrode on the lowest palpable rib. Placing the left arm electrode just above the left clavicle. Placing the right leg electrode on the lowest palpable rib. Placing the right arm electrode just below the right clavicle. p. 667 The nurse is reviewing the electrocardiography (ECG) strip after it is printed. How does the nurse interpret the data of the strip? <p>The nurse is reviewing the electrocardiography (ECG) strip after it is printed. How does the nurse interpret the data of the strip?</p> Speed is measured on the vertical axis. Time is measured on the horizontal axis. Five small blocks make up five large blocks. Vertical lines at the top represent 2-second segments. p. 667 The nurse analyzes a patient's electrocardiography (ECG) rhythm strip. During the analysis, the nurse determines that the patient has bradycardia and assesses for atrial regularity. Then the nurse measures the PR interval, the QRS duration, and identifies that the patient has ST depression. Which analyses must the nurse still perform to complete this ECG rhythm strip review? Select all that apply. <p>The nurse analyzes a patient's electrocardiography (ECG) rhythm strip. During the analysis, the nurse determines that the patient has bradycardia and assesses for atrial regularity. Then the nurse measures the PR interval, the QRS duration, and identifies that the patient has ST depression. Which analyses must the nurse still perform to complete this ECG rhythm strip review? <b>Select all that apply.</b> </p> Determining the heart rate Measuring the QT interval Observing the heart rhythm Examining the ST segment Noting the shape and height of the T wave Test-Taking Tip: Be alert for details about what you are being asked to do. In this Question Type, you are asked to select all options that apply to a given situation or patient. All options likely relate to the situation, but only some of the options may relate directly to the situation. p. 670 What is the ability of the cardiac cells to generate an electrical impulse spontaneously and repetitively called? <p>What is the ability of the cardiac cells to generate an electrical impulse spontaneously and repetitively called?</p> Excitability Conductivity Contractility Automaticity Rationale Automaticity is the ability of the cardiac cells to generate an electrical impulse spontaneously and repetitively. Excitability is the ability of the non-pacemaker heart cells to respond to an electrical impulse that begins in pacemaker cells. Conductivity is the ability to send an electrical stimulus from cell membrane to cell membrane. Contractility is the mechanical activity of the heart. Test-Taking Tip: Start by reading each of the answer options carefully. Usually at least one of them will be clearly wrong. Eliminate this one from consideration. Now you have reduced the number of response choices by one and improved the odds. Continue to analyze the options. If you can eliminate one more choice in a four-option question, you have reduced the odds to 50/50. While you are eliminating the wrong choices, recall often occurs. One of the options may serve as a trigger that causes you to remember what a few seconds ago had seemed completely forgotten. p. 665 What component of a normal electrocardiogram represents early ventricular repolarization? <p>What component of a normal electrocardiogram represents early ventricular repolarization?</p> U wave QT interval ST segment QRS complex p. 669 The nurse initiates cardiac monitoring for a patient with a potassium level of 2.5 mEq/L. What electrocardiogram (ECG) change does the nurse anticipate? <p>The nurse initiates cardiac monitoring for a patient with a potassium level of 2.5 mEq/L. What electrocardiogram (ECG) change does the nurse anticipate?</p> Absent P waves Inverted T wave Prolonged PR interval Premature atrial contractions Test-Taking Tip: Avoid spending excessive time on any one question. Most questions can be answered in 1 to 2 minutes. p. 669 Which is an expected finding for a normal PR interval? <p>Which is an expected finding for a normal PR interval?</p> 0.16 seconds 0.22 seconds 0.10 seconds 0.04 seconds p. 669 How should the nurse analyze the ventricular rhythm on an electrocardiogram (ECG) tracing? <p>How should the nurse analyze the ventricular rhythm on an electrocardiogram (ECG) tracing?</p> Measure the duration of the QRS complex. Assess the RR intervals throughout the ECG tracing. Evaluate the PP intervals throughout the ECG tracing. Count the number of QRS complexes in 6 seconds and multiply by 10. p. 715 The nurse is teaching a patient about self-management to prevent dysrhythmias caused by premature beats and ectopic rhythms. Which statement made by the patient indicates a need for further teaching? <p>The nurse is teaching a patient about self-management to prevent dysrhythmias caused by premature beats and ectopic rhythms. Which statement made by the patient indicates a need for further teaching?</p> "I have to quit smoking." "I have to limit alcohol intake." "I have to manage stress and avoid getting tired." "I have to drink coffee twice daily and other energy drinks to stay active." p. 682 What patients have a higher risk of developing infective endocarditis? Select all that apply. <p>What patients have a higher risk of developing infective endocarditis? <b>Select all that apply.</b> </p> Smokers IV drug users Patients with a cardiac defect Patients with thrombocytopenia Patients with a previous valve replacement Test-Taking Tip: Be alert for details. Details provided in the stem of the item, such as behavioral changes or clinical changes (or both) within a certain time period, can provide a clue to the most appropriate response or, in some cases, responses. p. 711 What patient population has the highest risk of being misdiagnosed when experiencing infective endocarditis? <p>What patient population has the <b>highest </b>risk of being misdiagnosed when experiencing infective endocarditis?</p> Infants Older adults Young children Pregnant women Test-Taking Tip: Avoid looking for an answer pattern or code. There may be times when four or five consecutive questions have the same letter or number for the correct answer. p. 711 A patient diagnosed with endocarditis develops peripheral edema, presents with weight gain, and reports a loss of appetite. What complication should the patient be evaluated for? <p>A patient diagnosed with endocarditis develops peripheral edema, presents with weight gain, and reports a loss of appetite. What complication should the patient be evaluated for?</p> Stroke Splenic infarction Right-sided heart failure Left-sided arterial embolization p. 711 A patient diagnosed with infective endocarditis has developed left-sided heart failure. What assessment findings are consistent with this complication? Select all that apply. <p>A patient diagnosed with infective endocarditis has developed left-sided heart failure. What assessment findings are consistent with this complication? <b>Select all that apply.</b> </p> Fatigue Anorexia Peripheral edema Shortness of breath Crackles in the lungs Test-Taking Tip: Read every word of each question and option before responding to the item. Glossing over the questions just to get through the examination quickly can cause you to misread or misinterpret the real intent of the question. p. 711 The patient diagnosed with infective endocarditis has gained 4 pounds in 24 hours and has peripheral edema. What complication should the nurse assess for? <p>The patient diagnosed with infective endocarditis has gained 4 pounds in 24 hours and has peripheral edema. What complication should the nurse assess for?</p> Splenic infarction Arterial embolism Right-sided heart failure Transient ischemic attack Test-Taking Tip: After you have eliminated one or more choices, you may discover that two of the options are very similar. This can be very helpful, because it may mean that one of these look-alike answers is the best choice and the other is a very good distractor. Test both of these options against the stem. Ask yourself which one completes the incomplete statement grammatically and which one answers the question more fully and completely. The option that best completes or answers the stem is the one you should choose. Here, too, pause for a few seconds, give your brain time to reflect, and recall may occur. p. 711 The nurse is assessing a patient diagnosed with infectious endocarditis for arterial embolization. What symptoms would indicate a splenic infarction? Select all that apply. <p>The nurse is assessing a patient diagnosed with infectious endocarditis for arterial embolization. What symptoms would indicate a splenic infarction? <b>Select all that apply.</b> </p> Chest pain Hematuria Abdominal pain Rebound tenderness Pain in the left shoulder Test-Taking Tip: If you can eliminate any responses as incorrect based on your knowledge, you will not be guessing randomly but will be exercising "informed guessing." p. 711 The nurse is caring for a patient diagnosed with infective endocarditis and knows that renal infarction is a comorbidity of infective endocarditis. Which symptoms, if present, would cause the nurse to suspect renal infarction? Select all that apply. <p>The nurse is caring for a patient diagnosed with infective endocarditis and knows that renal infarction is a comorbidity of infective endocarditis. Which symptoms, if present, would cause the nurse to suspect renal infarction? <b>Select all that apply.</b> </p> Pyuria Flank pain Hematuria Shortness of breath Abdominal distention Test-Taking Tip: What happens if you find yourself in a slump over the examination? Take a time-out to refocus and reenergize! Talk to friends and family who support your efforts in achieving one of your major accomplishments in life. This effort will help you regain confidence in yourself and get you back on track toward the realization of your long-anticipated goal. p. 711 A patient diagnosed with infective endocarditis develops abdominal pain with radiation to the left shoulder. What assessment is priority? Nail beds Lung sounds Mental status Rebound tenderness p. 711 The nurse is planning care for a patient who was just diagnosed with infective endocarditis. Which treatments does the nurse anticipate the provider to order? Select all that apply. <p>The nurse is planning care for a patient who was just diagnosed with infective endocarditis. Which treatments does the nurse anticipate the provider to order? <b>Select all that apply.</b> </p> Xenograft Valve repair IV antibiotics Anticoagulants Corticosteroids pp. 709-710 What teaching is priority for a patient currently undergoing treatment for bacterial endocarditis who is being discharged home? They need to avoid flossing their teeth They should know the daily schedule for home health nurses They need to know where to purchase the supplies needed for home They need to know the care of a peripherally inserted central catheter (PICC) line p. 712 The nurse provided teaching to a patient diagnosed with infective endocarditis. What statement by the patient indicates a need for further teaching? <p>The nurse provided teaching to a patient diagnosed with infective endocarditis. What statement by the patient indicates a need for further teaching?</p> "I will not use dental floss." "I will use antibiotic cream on any cuts." "I will make sure I take antibiotics anytime I go to the dentist." "I will call my health care provider if I have a fever or feel really tired." p. 712 The nurse is caring for a patient who is complaining of chest pain. Assessment findings include a scratchy, high-pitched sound over the patient's left side, at the fourth intercostal space. What lab value should the nurse evaluate? <p>The nurse is caring for a patient who is complaining of chest pain. Assessment findings include a scratchy, high-pitched sound over the patient's left side, at the fourth intercostal space. What lab value should the nurse evaluate?</p> Platelet count White blood cell count Hemoglobin and hematocrit Erythrocytes sedimentation rate (ESR) p. 713 A patient diagnosed with pericarditis reports severe substernal chest pain. What position should the patient be placed in? <p>A patient diagnosed with pericarditis reports severe substernal chest pain. What position should the patient be placed in?</p> Prone Supine Side-lying Upright, leaning forward p. 713 A patient is prescribed NSAIDs for pain related to pericarditis. What instructions are most important? <p>A patient is prescribed NSAIDs for pain related to pericarditis. What instructions are <b>most</b> important?</p> "Do not miss a dose." "Take the medication with food." "Avoid taking this medication at the same time as antibiotics." "Do not take this medication if body temperature is over 100.9° F." p. 713 Which medication would the nurse expect to see prescribed to reduce reoccurrence of pericarditis in a patient, following an initial diagnosis? <p>Which medication would the nurse expect to see prescribed to reduce reoccurrence of pericarditis in a patient, following an initial diagnosis?</p> Penicillin Ibuprofen Colchicine Metoprolol p. 713 The nurse is caring for a patient newly diagnosed with bacterial pericarditis. Which therapies does the nurse anticipate the provider will order? Select all that apply. <p>The nurse is caring for a patient newly diagnosed with bacterial pericarditis. Which therapies does the nurse anticipate the provider will order? <b>Select all that apply.</b> </p> Antibiotics Hemodialysis Corticosteroids Valve replacement Pericardial drainage A patient suspected of having cardiac tamponade is scheduled for an echocardiogram. Which intervention should be done first? <p>A patient suspected of having cardiac tamponade is scheduled for an echocardiogram. Which intervention should be done <b>first</b>?</p> Repeat vital signs Administer digoxin Administer IV fluids Draw blood for a complete blood count (CBC) p. 713 What are common manifestations of rheumatic carditis? Select all that apply. <p>What are common manifestations of rheumatic carditis? <b>Select all that apply.</b> </p> Bradycardia Cardiomegaly Atrial fibrillation Change in an existing murmur History of a streptococcal infection p. 714 A patient with a history of rheumatic carditis presents to the emergency department with a red sore throat and the following vital signs: blood pressure 130/80, pulse 112, respiratory rate 20, oxygen saturation 95% on room air, and a temperature of 102.3°F. What nursing assessment is priority? <p>A patient with a history of rheumatic carditis presents to the emergency department with a red sore throat and the following vital signs: blood pressure 130/80, pulse 112, respiratory rate 20, oxygen saturation 95% on room air, and a temperature of 102.3&#xb0;F. What nursing assessment is <b>priority</b>?</p> Heart sounds Throat culture Echocardiogram Labs for white blood cell count p. 714 A patient diagnosed with rheumatic carditis is being seen for a follow-up after antibiotic therapy. What finding should be reported to the health care practitioner first? <p>A patient diagnosed with rheumatic carditis is being seen for a follow-up after antibiotic therapy. What finding should be reported to the health care practitioner <b>first</b>?</p> Fever Pulse of 100 Pericardial friction rub Reddened throat with exudate p. 714 The nurse received a report on four patients. What patient should the nurse see first? <p>The nurse received a report on four patients. What patient should the nurse see <b>first</b>?</p> A patient with pericarditis who has muffled heart sounds A patient with rheumatic carditis who has a fever of 101.3° F A patient with infective endocarditis who has an audible murmur A patient who underwent valve replacement surgery 24 hours ago p. 714 A patient received teaching regarding recurrence of endocarditis. What statement by the patient indicates a need for further teaching? <p>A patient received teaching regarding recurrence of endocarditis. What statement by the patient indicates a need for further teaching?</p> "I will need to record my temperature for the next 6 weeks." "I should call my primary care provider if I have fever or chills." "Once I have been treated, I will not develop endocarditis again." "I will only need antibiotics if I have an invasive dental procedure." p. 712 The emergency department nurse is performing an admission assessment on a patient who just presented to the emergency department. Which assessment findings, if present in the patient, would prompt the nurse to suspect a left-sided arterial embolization? Select all that apply. <p>The emergency department nurse is performing an admission assessment on a patient who just presented to the emergency department. Which assessment findings, if present in the patient, would prompt the nurse to suspect a left-sided arterial embolization? <b>Select all that apply.</b> </p> Confusion Flank pain Chest pain Shortness of breath Rebound tenderness p. 711 When assessing a patient diagnosed with infective endocarditis, the nurse notes petechiae around the clavicles and splinter hemorrhages of the nail beds. What action should the nurse perform first? <p>When assessing a patient diagnosed with infective endocarditis, the nurse notes petechiae around the clavicles and splinter hemorrhages of the nail beds. What action should the nurse perform <b>first</b>?</p> Document the findings Contact the health care provider Determine if there is a pleural friction rub Check the patient's throat for redness and exudate p. 711 The emergency department nurse is performing an admission assessment on a patient who just presented to the emergency department. Which assessment findings, if present in the patient, would prompt the nurse to suspect a right-sided arterial embolization? Select all that apply. <p>The emergency department nurse is performing an admission assessment on a patient who just presented to the emergency department. Which assessment findings, if present in the patient, would prompt the nurse to suspect a right-sided arterial embolization? <b>Select all that apply.</b> </p> Confusion Flank pain Chest pain Shortness of breath Rebound tenderness p. 711 The nurse is caring for a patient with endocarditis. Which assessment findings, if present in the patient, would prompt the nurse to suspect right-sided heart failure secondary to endocarditis? Select all that apply. <p>The nurse is caring for a patient with endocarditis. Which assessment findings, if present in the patient, would prompt the nurse to suspect right-sided heart failure secondary to endocarditis? <b>Select all that apply.</b> </p> Fatigue Weight gain Peripheral edema Shortness of breath Crackles when auscultating lungs p. 711 The nurse is planning care for a patient with infective endocarditis. The nurse uses which outcomes to determine when the patient is ready to be discharged? Select all that apply. <p>The nurse is planning care for a patient with infective endocarditis. The nurse uses which outcomes to determine when the patient is ready to be discharged? <b>Select all that apply.</b> </p> Decreased pyuria No peripheral edema Negative blood cultures Temperatures lower than 100.5° F Completion of ten days of antibiotics p. 712 The nurse is caring for a patient with pericarditis. Which medicines, if ordered for the patient, would the nurse administer as ordered? Select all that apply. <p>The nurse is caring for a patient with pericarditis. Which medicines, if ordered for the patient, would the nurse administer as ordered? <b>Select all that apply.</b> </p> Aspirin Ibuprofen Colchicine Antibiotics Anticoagulants p. 713 The nurse is caring for a patient with pericarditis. Which assessment findings, if present in the patient, would prompt the nurse to suspect cardiac tamponade? Select all that apply. <p>The nurse is caring for a patient with pericarditis. Which assessment findings, if present in the patient, would prompt the nurse to suspect cardiac tamponade? <b>Select all that apply.</b> </p> Tachycardia Atrial fibrillation Pulsus paradoxus Jugular vein distention Decreased cardiac output p. 713 The nurse is preparing to assess a patient diagnosed with rheumatic carditis. Which assessment findings does the nurse expect to see in this patient? Select all that apply. <p>The nurse is preparing to assess a patient diagnosed with rheumatic carditis. Which assessment findings does the nurse expect to see in this patient? <b>Select all that apply.</b> </p> Cardiomegaly Precordial pain Heart rate of 58 Shortened PR interval Streptococcal infection p. 714 A patient diagnosed with streptococcal pharyngitis would be prescribed which antibiotic to prevent the development of rheumatic carditis? Select all that apply. <p>A patient diagnosed with streptococcal pharyngitis would be prescribed which antibiotic to prevent the development of rheumatic carditis? <b>Select all that apply.</b> </p> Penicillin Macrolides Erythromycin Cephalosporin Fluoroquinolones p. 714 What finding is consistent with effective pericardial drainage during a pericardiocentesis? <p>What finding is consistent with effective pericardial drainage during a pericardiocentesis?</p> Decreased heart rate Decreased heart sounds Decreased right atrial pressure Increased pulmonary artery pressure p. 713 What procedure will allow excessive pericardial fluid to drain into the pleural space? <p>What procedure will allow excessive pericardial fluid to drain into the pleural space?</p> Pericardiectomy Pericardial shunt Pericardiocentesis Pericardial window p. 714 The nurse is caring for four patients who are all diagnosed with rheumatic carditis. What patient should the nurse see first? <p>The nurse is caring for four patients who are all diagnosed with rheumatic carditis. What patient should the nurse see <b>first</b>?</p> The patient with urticaria The patient with cardiomegaly The patient with aortic stenosis The patient with mitral regurgitation p. 714 The patient presents to the emergency department with an enlarged heart and a new murmur. The vital signs are blood pressure 108/55, pulse 118, oxygen saturation 94% on room air, and a temperature of 103.4° F. When gathering the health history, what question by the nurse is a priority? <p>The patient presents to the emergency department with an enlarged heart and a new murmur. The vital signs are blood pressure 108/55, pulse 118, oxygen saturation 94% on room air, and a temperature of 103.4&#xb0; F. When gathering the health history, what question by the nurse is a <b>priority</b>?</p> "Are you currently taking NSAIDs?" "Have you recently had a sore throat?" "Have you recently had a dental cleaning?" "Did you receive all of your childhood immunizations?" p. 714 The patient diagnosed with rheumatic fever is receiving education about antibiotics. What teaching is most important? <p>The patient diagnosed with rheumatic fever is receiving education about antibiotics. What teaching is <b>most</b> important?</p> "Antibiotics should be taken with food." "All of the medication should be taken." "The medication may cause nausea or diarrhea." "If a dose is missed, it should be taken with the next scheduled dose." p. 714 A patient diagnosed with infective endocarditis is scheduled to have a valve replacement. What information is most important to report to the surgeon? <p>A patient diagnosed with infective endocarditis is scheduled to have a valve replacement. What information is <b>most</b> important to report to the surgeon?</p> Petechiae Dental caries An audible murmur Splinter hemorrhages p. 709 Which assessment or test is used to confirm a diagnosis of infective endocarditis? <p>Which assessment or test is used to confirm a diagnosis of infective endocarditis?</p> Echocardiogram Complete blood count Positive blood cultures Presence of third heart sound p. 711 A patient prescribed ibuprofen 24 hours ago for pericarditis asks the nurse why the pain has not improved. What statement by the nurse is most appropriate? <p>A patient prescribed ibuprofen 24 hours ago for pericarditis asks the nurse why the pain has not improved. What statement by the nurse is <b>most</b> appropriate?</p> "You may not be taking enough ibuprofen." "It may take another day before you have pain relief." "You should make sure you're taking the ibuprofen with food." "The ibuprofen will not help with the pain; it is prescribed to decrease inflammation." p. 713 What is a definitive treatment for constrictive pericarditis? <p>What is a definitive treatment for constrictive pericarditis?</p> Antibiotics Pericardiectomy Pericardiocentesis Pericardial window p. 713 The patient is being monitored after having a pericardiocentesis for treatment of cardiac tamponade. When assessing the patient, the nurse notes muffled heart sounds and a blood pressure reading of 84/50. What action by the nurse is priority? <p>The patient is being monitored after having a pericardiocentesis for treatment of cardiac tamponade. When assessing the patient, the nurse notes muffled heart sounds and a blood pressure reading of 84/50. What action by the nurse is <b>priority</b>?</p> Decrease the IV fluids Prepare for an emergency sternotomy Prepare for a repeat pericardiocentesis Administer prescribed pain medication pp. 713-714 A patient diagnosed with pericarditis is being evaluated for pulsus paradoxus. Which of the following is consistent with this diagnosis? <p>A patient diagnosed with pericarditis is being evaluated for pulsus paradoxus. Which of the following is consistent with this diagnosis?</p> A blood pressure of 140/70 An apical pulse of 90 and a radial pulse of 75 A brachial pulse of 110 and a diastolic blood pressure of 55 A systolic pressure of 115 on expiration and a systolic pressure of 100 on inspiration p. 713 What treatment is indicated for a patient diagnosed with uremic pericarditis? <p>What treatment is indicated for a patient diagnosed with uremic pericarditis?</p> Radiation Antibiotics Hemodialysis Pericardial window p. 713 What information should the nurse include when providing teaching to a patient diagnosed with endocarditis? Select all that apply. <p>What information should the nurse include when providing teaching to a patient diagnosed with endocarditis? <b>Select all that apply.</b> </p> Reporting signs of heart failure Balancing activity with adequate rest Importance of follow-up blood cultures Completing 10 days of antibiotic therapy The need for antibiotic prophylaxis any time the patient goes to the dentist A patient has been diagnosed with acute pericarditis. On continuous electrocardiography, the patient has an irregular heart rate of 110 with no discernible P waves. This is consistent with what common cardiac rhythm found in patients with this diagnosis? <p>A patient has been diagnosed with acute pericarditis. On continuous electrocardiography, the patient has an irregular heart rate of 110 with no discernible P waves. This is consistent with what common cardiac rhythm found in patients with this diagnosis?</p> Atrial fibrillation Sinus tachycardia Ventricular tachycardia Second-degree heart block p. 713 The nurse is caring for a patient diagnosed with acute pericarditis. What symptoms are consistent with this diagnosis? Select all that apply. <p>The nurse is caring for a patient diagnosed with acute pericarditis. What symptoms are consistent with this diagnosis? <b>Select all that apply.</b> </p> Hypotension Pain on inspiration Substernal chest pain Muffled heart sounds Jugular vein distention p. 713 The nurse is caring for four patients. What patient should the nurse see first? <p>The nurse is caring for four patients. What patient should the nurse see <b>first</b>?</p> A patient diagnosed with pericarditis with muffled heart sounds A patient admitted with rheumatic endocarditis who has developed peripheral edema A patient with infective endocarditis scheduled for a valve replacement surgery today A patient admitted with a diagnosis of infective endocarditis with a temperature of 100.3° F pp. 712-713 A patient presents to the emergency department with a fever of 102.3° F after being discharged after an appendectomy. What physical assessment is most important to rule out infective endocarditis? <p>A patient presents to the emergency department with a fever of 102.3&#xb0; F after being discharged after an appendectomy. What physical assessment is <b>most</b> important to rule out infective endocarditis?</p> Lung sounds Heart sounds Mental status Peripheral pulses Test-Taking Tip: If the question asks for an immediate action or response, all of the answers may be correct, so base your selection on identified priorities for action. p. 711 Which patient has the highest risk for developing infective endocarditis? <p>Which patient has the <b>highest</b> risk for developing infective endocarditis?</p> A 16-year-old girl with a cardiac defect diagnosed with scabies A 24-year-old patient with a recent history of systemic infection who is being seen for preoperative care A 65-year-old patient with a history of a valve replacement diagnosed with chronic renal failure A 45-year-old patient with a history of IV drug use who recently had a dental procedure p. 711 What are possible causes of infective endocarditis? Select all that apply. <p>What are possible causes of infective endocarditis? <b>Select all that apply.</b> </p> Stroke Renal failure Skin infection IV-line placement Urinary tract infection p. 711 A patient presents to the emergency department with weight loss, petechiae, and splinter hemorrhages. The patient's vital signs are blood pressure 135/82, pulse 108, oxygen saturation 96%, on room air, and temperature 101.4° F. Upon physical assessment, a new murmur is noted. What diagnostic test should be done first? <p>A patient presents to the emergency department with weight loss, petechiae, and splinter hemorrhages. The patient's vital signs are blood pressure 135/82, pulse 108, oxygen saturation 96%, on room air, and temperature 101.4&#xb0; F. Upon physical assessment, a new murmur is noted. What diagnostic test should be done <b>first</b>?</p> CT scan Chest x-ray Blood cultures Echocardiogram Test-Taking Tip: Avoid looking for an answer pattern or code. There may be times when four or five consecutive questions have the same letter or number for the correct answer. p. 711 A patient diagnosed with pericarditis has been treated for 3 days. The patient reports no pain relief 40 minutes after the administration of NSAIDs. What action by the nurse is most appropriate? <p>A patient diagnosed with pericarditis has been treated for 3 days. The patient reports no pain relief 40 minutes after the administration of NSAIDs. What action by the nurse is <b>most</b> appropriate?</p> Contact the health care provider Perform a complete cardiac assessment Encourage the patient to sit up and forward Reassure the patient that the pain should get better soon p. 713 A patient diagnosed with infectious endocarditis is confused and is having difficulty speaking. What action should the nurse perform first? <p>A patient diagnosed with infectious endocarditis is confused and is having difficulty speaking. What action should the nurse perform <b>first</b>?</p> Administer oxygen Contact the health care provider Perform a full mental status exam Place the patient in high-Fowler's position p. 711 While assessing an older adult patient with a history of infective endocarditis and a valve replacement, a new murmur is noted. What actions are most important? Select all that apply. <p>While assessing an older adult patient with a history of infective endocarditis and a valve replacement, a new murmur is noted. What actions are <b>most </b>important? <b>Select all that apply.</b> </p> Obtain a temperature Perform a skin assessment Draw a set of blood cultures Order a stat echocardiogram Contact the health care provider Test-Taking Tip: Sometimes the reading of a question in the middle or toward the end of an exam may trigger your mind with the answer or provide an important clue to an earlier question. p. 711 The nurse is caring for a patient with pericarditis. Which interventions are appropriate to be included in the patient's plan of care? Select all that apply. <p>The nurse is caring for a patient with pericarditis. Which interventions are appropriate to be included in the patient's plan of care? <b>Select all that apply.</b> </p> Administer aspirin Monitor cardiac output Administer an anti-inflammatory agent Avoid administering anticoagulant agents Instruct the patient to avoid leaning forward p. 713 The nurse is assessing a patient with a cardiac infection. Which symptoms support the diagnosis of infective endocarditis instead of pericarditis or rheumatic carditis? <p>The nurse is assessing a patient with a cardiac infection. Which symptoms support the diagnosis of infective endocarditis instead of pericarditis or rheumatic carditis?</p> Splinter hemorrhages Thickening of the endocardium Friction rub auscultated at the left lower sternal border Pain aggravated by breathing, coughing, and swallowing p. 711 The nurse reviews the medical record of a patient who is diagnosed with rheumatic carditis and notes a history of penicillin sensitivity. The nurse anticipates that which medication will be prescribed for this patient? <p>The nurse reviews the medical record of a patient who is diagnosed with rheumatic carditis and notes a history of penicillin sensitivity. The nurse anticipates that which medication will be prescribed for this patient?</p> Warfarin Amiodarone Erythromycin Levosimendan p. 714 Which factors are associated with the development of infective endocarditis? Select all that apply. <p>Which factors are associated with the development of infective endocarditis? <b>Select all that apply.</b> </p> Sepsis Heart failure Pulmonary embolism Myocardial infarction Cystoscopy 2 days ago Intravenous heroin use p. 711 For which cardiac conditions is antimicrobial therapy most likely to be used? Select all that apply. <p>For which cardiac conditions is antimicrobial therapy most likely to be used? <b>Select all that apply.</b> </p> Heart failure Cardiomyopathy Rheumatic carditis Infective endocarditis Valvular heart disease pp. 708, 712, 714 What medications are contraindicated for patients diagnosed with nonobstructed hypertrophic cardiomyopathy? <p>What medications are contraindicated for patients diagnosed with nonobstructed hypertrophic cardiomyopathy?</p> Nitrates Diuretics Beta blockers Calcium channel blockers Test-Taking Tip: Avoid looking for an answer pattern or code. There may be times when four or five consecutive questions have the same letter or number for the correct answer. p. 715 What medication can be administered to control the heart rate of a patient diagnosed with obstructed hypertrophic cardiomyopathy? <p>What medication can be administered to control the heart rate of a patient diagnosed with obstructed hypertrophic cardiomyopathy?</p> Apixaban Metoprolol Nitroglycerin Isosorbide dinitrate Test-Taking Tip: Avoid looking for an answer pattern or code. There may be times when four or five consecutive questions have the same letter or number for the correct answer. p. 716 A patient undergoing chemotherapy for breast cancer is being seen in the emergency department for dyspnea, fatigue, and palpitations. The patient is noted to be in a second-degree heart block and has an elevated brain natriuretic peptide level. What additional finding would indicate a diagnosis of dilated cardiomyopathy? <p>A patient undergoing chemotherapy for breast cancer is being seen in the emergency department for dyspnea, fatigue, and palpitations. The patient is noted to be in a second-degree heart block and has an elevated brain natriuretic peptide level. What additional finding would indicate a diagnosis of dilated cardiomyopathy?</p> Dilated ventricles Small chamber size Hypertrophy of the septum Fibrous fatty myocardial tissue pp. 714-716 What factor is most important when obtaining a health history for a patient with enlarged ventricles? <p>What factor is <b>most</b> important when obtaining a health history for a patient with enlarged ventricles?</p> Alcohol use Athletic activities Recent use of beta blockers Family history of cardiomyopathy p. 714 What findings are common in a patient with nonobstructed hypertrophic cardiomyopathy? Select all that apply. <p>What findings are common in a patient with nonobstructed hypertrophic cardiomyopathy? <b>Select all that apply.</b> </p> Syncope Dyspnea Heart block Mitral valve murmur Pulmonary embolism p. 715 The nurse is caring for a woman undergoing fertility treatments. The woman's teenage son recently died from hypertrophic cardiomyopathy while playing basketball. What action by the nurse is most important? <p>The nurse is caring for a woman undergoing fertility treatments. The woman's teenage son recently died from hypertrophic cardiomyopathy while playing basketball. What action by the nurse is <b>most</b> important?</p> Recommend genetic counseling Encourage the patient to discuss her feelings Suggest 3-D ultrasonography throughout the pregnancy Provide reassurance that this is unlikely to happen again p. 716 What patients have increased risk for dilated cardiomyopathy? Select all that apply. <p>What patients have increased risk for dilated cardiomyopathy? <b>Select all that apply.</b> </p> Patients with poor nutrition Patients with excessive alcohol use Patients who are involved in athletics Patients who recently had chemotherapy Patients who have family members with cardiomyopathy p. 714 What diseases are common causes of restrictive cardiomyopathy? Select all that apply. <p>What diseases are common causes of restrictive cardiomyopathy? <b>Select all that apply.</b> </p> Infection Sarcoidosis Amyloidosis Alcohol abuse Chemotherapy pp. 714-715 Which patient should the nurse see first? <p>Which patient should the nurse see <b>first</b>?</p> A patient with restrictive cardiomyopathy who has had a rhythm change A patient with a dilated cardiomyopathy who has crackles on auscultation A patient with obstructed hypertrophic cardiomyopathy with audible mitral valve murmur A patient diagnosed with nonobstructive hypertrophic cardiomyopathy reporting dyspnea pp. 714-715 What medication should be given first to a patient diagnosed with obstructed hypertrophic cardiomyopathy and secondary atrial fibrillation? <p>What medication should be given <b> first </b>to a patient diagnosed with obstructed hypertrophic cardiomyopathy and secondary atrial fibrillation?</p> Adenosine Metoprolol Nitroglycerin Isosorbide dinitrate p. 716 What statement by a patient diagnosed with dilated cardiomyopathy indicates a need for further teaching? <p>What statement by a patient diagnosed with dilated cardiomyopathy indicates a need for further teaching?</p> "I need to space out my activities." "There is no treatment for this type of cardiomyopathy." "My shortness of breath is probably from my heart failure." "I have to make sure I take my isosorbide dinitrate every day." Test-Taking Tip: Look for options that are similar in nature. If all are correct, either the question is poor or all options are incorrect, the latter of which is more likely. Example: If the answer you are seeking is directed to a specific treatment and all but one option deal with signs and symptoms, you would be correct in choosing the treatment-specific option. p. 715 What test will confirm a diagnosis of ventricular dilation secondary to dilated cardiomyopathy? <p>What test will confirm a diagnosis of ventricular dilation secondary to dilated cardiomyopathy?</p> Chest x-ray Troponin level Electrocardiogram (ECG) Brain natriuretic peptide (BNP) Test-Taking Tip: Identify option components as correct or incorrect. This may help you identify a wrong answer. p. 697 What statement by the patient regarding dilated cardiomyopathy indicates a need for further teaching? <p>What statement by the patient regarding dilated cardiomyopathy indicates a need for further teaching?</p> "I probably got this as a result of excessive alcohol use." "I may need to sleep with my head up higher than normal." "I am probably going to always have an irregular heartbeat." "I'm not going to be able to do as much exercise as I used to do." pp. 714-716 A 22-year-old male patient presents to the emergency department with severe chest pain that began while he was home with his family watching TV. The patient has had three sublingual nitroglycerin tabs with no relief of chest pain. What action by the nurse is priority? <p>A 22-year-old male patient presents to the emergency department with severe chest pain that began while he was home with his family watching TV. The patient has had three sublingual nitroglycerin tabs with no relief of chest pain. What action by the nurse is <b>priority</b>?</p> Prepare for immediate cardioversion Prepare the patient for echocardiography Administer an additional nitroglycerin tab Determine if there is a family history of cardiomyopathy p. 715 What medications does the nurse anticipate administering to a patient newly diagnosed with restrictive cardiomyopathy? Select all that apply. <p>What medications does the nurse anticipate administering to a patient newly diagnosed with restrictive cardiomyopathy? <b>Select all that apply.</b> </p> Steroids Diuretics Antibiotics Vasodilators Cardiac glycosides pp. 715-716 What medications does the nurse plan to administer to a patient diagnosed with obstructive hypertrophic cardiomyopathy? Select all that apply. <p>What medications does the nurse plan to administer to a patient diagnosed with obstructive hypertrophic cardiomyopathy? <b>Select all that apply.</b> </p> Nitrates Diuretics Vasodilators Calcium antagonists Beta adrenergic blocking agents p. 716 The nurse is caring for a patient diagnosed with hypertrophic cardiomyopathy who is complaining of chest pain. The patient stops speaking, and the cardiac monitor shows ventricular tachycardia. What action should the nurse take first? <p>The nurse is caring for a patient diagnosed with hypertrophic cardiomyopathy who is complaining of chest pain. The patient stops speaking, and the cardiac monitor shows ventricular tachycardia. What action should the nurse take <b>first</b>?</p> Call for help Check the pulse Perform a sternal rub Begin cardiopulmonary resuscitation pp. 705, 716 A patient presents to the emergency department with sudden onset of palpitations, crackles in the lungs, and severe dyspnea on exertion. What type of cardiomyopathy does the nurse suspect? <p>A patient presents to the emergency department with sudden onset of palpitations, crackles in the lungs, and severe dyspnea on exertion. What type of cardiomyopathy does the nurse suspect?</p> Dilated cardiomyopathy Restrictive cardiomyopathy Hypertrophic cardiomyopathy Arrhythmogenic right ventricular cardiomyopathy p. 715 A patient diagnosed with dilated cardiomyopathy is suspected of having right-sided heart failure. What assessment finding does the nurse expect? <p>A patient diagnosed with dilated cardiomyopathy is suspected of having right-sided heart failure. What assessment finding does the nurse expect?</p> Dyspnea Orthopnea Dependent edema Crackles in the lungs Test-Taking Tip: The most reliable way to ensure that you select the correct response to a multiple-choice question is to recall it. Depend on your learning and memory to furnish the answer to the question. To do this, read the stem, and then stop! Do not look at the response options yet. Try to recall what you know and, based on this, what you would give as the answer. After you have taken a few seconds to do this, then look at all of the choices and select the one that most nearly matches the answer you recalled. It is important that you consider all the choices and not just choose the first option that seems to fit the answer you recall. Remember the distractors. The second choice may look okay, but the fourth choice may be worded in a way that makes it a slightly better choice. If you do not weigh all the choices, you are not maximizing your chances of correctly answering each question. p. 695 What medications would the nurse anticipate administering to a patient diagnosed with dilated cardiomyopathy? Select all that apply. <p>What medications would the nurse anticipate administering to a patient diagnosed with dilated cardiomyopathy? <b>Select all that apply.</b> </p> Digoxin Adenosine Magnesium Furosemide Nitroglycerin pp. 715-716 Which medication is indicated for a patient diagnosed with restrictive cardiomyopathy to block inappropriate sympathetic stimulation and tachycardia? <p>Which medication is indicated for a patient diagnosed with restrictive cardiomyopathy to block inappropriate sympathetic stimulation and tachycardia?</p> Digoxin Diltiazem Metoprolol Nitroglycerin p. 716 What statement by a patient diagnosed with obstructive hypertrophic cardiomyopathy reflects a need for further teaching? <p>What statement by a patient diagnosed with obstructive hypertrophic cardiomyopathy reflects a need for further teaching?</p> "I will not be able to take diuretics anymore." "The carvedilol should decrease the number of times I pass out." "I may be switched back to metoprolol to help increase my pulse rate." "I still won't be able to do strenuous exercise even though I am taking diltiazem." pp. 715-716 A patient scheduled for a heart transplant is being evaluated prior to the procurement of the donor heart. What finding should be reported immediately to the health care provider? <p>A patient scheduled for a heart transplant is being evaluated prior to the procurement of the donor heart. What finding should be reported <b>immediately</b> to the health care provider?</p> Temperature of 99.0° F A blood pressure of 140/83 White blood cell count of 10,500 A decrease in pulmonary vascular resistance p. 716 What negative inotropic agent can be administered to a patient with obstructive hypertrophic cardiomyopathy? <p>What negative inotropic agent can be administered to a patient with obstructive hypertrophic cardiomyopathy?</p> Digoxin Metoprolol Nitroglycerin Hydrochlorothiazide Test-Taking Tip: After you have eliminated one or more choices, you may discover that two of the options are very similar. This can be very helpful, because it may mean that one of these look-alike answers is the best choice and the other is a very good distractor. Test both of these options against the stem. Ask yourself which one completes the incomplete statement grammatically and which one answers the question more fully and completely. The option that best completes or answers the stem is the one you should choose. Here, too, pause for a few seconds, give your brain time to reflect, and recall may occur. p. 716 A patient with arrhythmogenic right ventricular cardiomyopathy has not responded to drug therapy. What treatment does the nurse anticipate preparing the patient for? <p>A patient with arrhythmogenic right ventricular cardiomyopathy has not responded to drug therapy. What treatment does the nurse anticipate preparing the patient for?</p> Ablation Defibrillation Heart transplant Synchronized cardioversion p. 716 What surgical procedure is a treatment for patients diagnosed with obstructive hypertrophic cardiomyopathy? <p>What surgical procedure is a treatment for patients diagnosed with obstructive hypertrophic cardiomyopathy?</p> Heart transplant Internal defibrillator Ventriculomyomectomy Radio frequency catheter ablation Test-Taking Tip: Sometimes the reading of a question in the middle or toward the end of an exam may trigger your mind with the answer or provide an important clue to an earlier question. p. 716 The nurse is assessing the cardiac rhythm for a patient who has just undergone a heart transplant. Two unrelated P waves are visible on the ECG rhythm. What action should the nurse take first? <p>The nurse is assessing the cardiac rhythm for a patient who has just undergone a heart transplant. Two unrelated P waves are visible on the ECG rhythm. What action should the nurse take <b>first</b>?</p> Increase the oxygen Document the finding Prepare for cardioversion Contact the health care provider p. 716 What assessment finding should be reported to the health care provider immediately if found in a postoperative heart transplant patient? <p>What assessment finding should be reported to the health care provider immediately if found in a postoperative heart transplant patient?</p> Two P waves on ECG Muffled heart sounds Orthostatic hypotension White blood cell count of 9000 p. 716 What medication is needed to support heart rate and maintain cardiac output in a patient who has had a heart transplant? <p>What medication is needed to support heart rate and maintain cardiac output in a patient who has had a heart transplant?</p> Digoxin Atropine Adenosine Isoproterenol p.. 716 A patient who has just undergone heart transplant surgery reports feeling lightheaded when changing positions. What action should the nurse take first? <p>A patient who has just undergone heart transplant surgery reports feeling lightheaded when changing positions. What action should the nurse take <b>first</b>?</p> Order a stat 12-lead ECG Administer isoproterenol Contact the health care provider Check orthostatic blood pressures p. 716 A patient who has undergone heart transplant surgery has developed shortness of breath. Vital signs are blood pressure 85/48, pulse 52, and oxygen saturation 92% on 2 L O 2 via nasal cannula. What action should the nurse take first? <p>A patient who has undergone heart transplant surgery has developed shortness of breath. Vital signs are blood pressure 85/48, pulse 52, and oxygen saturation 92% on 2 L O <sub>2 </sub>via nasal cannula. What action should the nurse take <b>first</b>?</p> Contact the health care provider Administer immunosuppressants Increase the oxygen concentration Place the patient in supine position p. 716 What type of medication is prescribed for a patient who has undergone heart transplant surgery? <p>What type of medication is prescribed for a patient who has undergone heart transplant surgery?</p> Atropine Adenosine Corticosteroids Immunosuppressants p. 716 What types of cardiomyopathy can be treated with heart transplantation? Select all that apply. <p>What types of cardiomyopathy can be treated with heart transplantation? <b>Select all that apply.</b> </p> Dilated cardiomyopathy Restrictive cardiomyopathy Obstructed hypertrophic cardiomyopathy Nonobstructed hypertrophic cardiomyopathy Arrhythmogenic right ventricular cardiomyopathy p. 716 What procedure does the nurse prepare the patient for if the surgeon is trying to rule out rejection of a new heart transplant? <p>What procedure does the nurse prepare the patient for if the surgeon is trying to rule out rejection of a new heart transplant?</p> Echocardiogram Nuclear stress test Endomyocardial biopsy Computed tomography scan p. 716 What medication may be prescribed to a patient at risk for development of coronary artery vasculopathy to prevent coronary spasm and closure? <p>What medication may be prescribed to a patient at risk for development of coronary artery vasculopathy to prevent coronary spasm and closure?</p> Digoxin Diltiazem Metoprolol Nitroglycerin p. 717 What tests are scheduled in order to identify coronary artery vasculopathy in patients who have undergone heart transplant? Select all that apply. <p>What tests are scheduled in order to identify coronary artery vasculopathy in patients who have undergone heart transplant? <b>Select all that apply.</b> </p> Angiography Echocardiogram Nuclear stress test Exercise tolerance test Endomyocardial biopsy p. 717 A 68-year-old patient has developed symptoms of right- and left-sided heart failure secondary to restrictive cardiomyopathy. What action by the nurse is priority? <p>A 68-year-old patient has developed symptoms of right- and left-sided heart failure secondary to restrictive cardiomyopathy. What action by the nurse is <b>priority</b>?</p> Discuss hospice care Prepare the patient for heart transplant Give the patient education about antidysrhythmic drugs Provide information about implantable cardiac defibrillators pp. 701, 716 In what age group is hypertrophic cardiomyopathy commonly diagnosed? <p>In what age group is hypertrophic cardiomyopathy commonly diagnosed?</p> Older adults Young adults Middle-aged adults School-aged children p. 714 A patient diagnosed with dilated cardiomyopathy has developed atrial fibrillation. For which complication should the nurse assess the patient? <p>A patient diagnosed with dilated cardiomyopathy has developed atrial fibrillation. For which complication should the nurse assess the patient?</p> Syncope Hemorrhage Renal failure Pulmonary embolism p. 715 The nurse is caring for a patient with obstructive hypertrophic cardiomyopathy. Which medications, if ordered for the patient by the provider, would prompt the nurse to contact the provider prior to administration? Select all that apply. <p>The nurse is caring for a patient with obstructive hypertrophic cardiomyopathy. Which medications, if ordered for the patient by the provider, would prompt the nurse to contact the provider prior to administration? <b>Select all that apply.</b> </p> Nitrates Diuretics Vasodilators Beta blockers Calcium antagonist pp. 715-716 The nurse is providing teaching to a patient after heart transplant surgery. What statement by the patient indicates a need for further teaching? <p>The nurse is providing teaching to a patient after heart transplant surgery. What statement by the patient indicates a need for further teaching?</p> "It is important to avoid people who are sick." "Handwashing is a very important part of avoiding infections." "I should avoid taking immunosuppressive drugs when I am sick." "Infection is a major cause of death in patients who have had the surgery." p. 716 What health care professional should be consulted when planning exercise for a patient who had heart transplant surgery? <p>What health care professional should be consulted when planning exercise for a patient who had heart transplant surgery?</p> Case manager Cardiac surgeon Physical therapist Discharge planner p. 717 What teaching is most important for a patient diagnosed with restrictive cardiomyopathy? <p>What teaching is <b>most</b> important for a patient diagnosed with restrictive cardiomyopathy?</p> Avoidance of sick people Importance of a balanced diet Reporting palpitations or dizziness Adherence to a medication regimen The nurse working as a transplant coordinator knows that each candidate must meet which criteria to ensure compatibility of the transplanted heart? Select all that apply. <p>The nurse working as a transplant coordinator knows that each candidate must meet which criteria to ensure compatibility of the transplanted heart? <b>Select all that apply.</b> </p> Same sex Similar height ABO compatibility Age within 10 years Comparable body weight p. 716 What complication is often concealed and should be assessed for in any patient who has had heart transplant surgery? <p>What complication is often concealed and should be assessed for in any patient who has had heart transplant surgery?</p> Pain Dysrhythmias Postoperative bleeding Excessive vagal stimulation p. 716 The nurse is caring for a patient diagnosed with obstructive hypertrophic cardiomyopathy. Which medications, if ordered by the provider, would prompt the nurse to hold the medication and contact the provider immediately? Select all that apply. <p>The nurse is caring for a patient diagnosed with obstructive hypertrophic cardiomyopathy. Which medications, if ordered by the provider, would prompt the nurse to hold the medication and contact the provider immediately? <b>Select all that apply.</b> </p> Diuretics Nitrates Cardiac glycosides Calcium antagonists Beta-adrenergic blockers p. 716 The patient, a college athlete who collapsed during soccer practice, has been diagnosed with hypertrophic cardiomyopathy. The patient says, "This can't be. I am in great shape. I eat right and exercise." What is the nurse's best response? <p>The patient, a college athlete who collapsed during soccer practice, has been diagnosed with hypertrophic cardiomyopathy. The patient says, "This can't be. I am in great shape. I eat right and exercise." What is the nurse's <b>best</b> response?</p> "How does this make you feel?" "This may be caused by a genetic trait." "This can be caused by taking performance-enhancing drugs." "Just imagine how bad it would be if you weren't in good shape." Test-Taking Tip: If you are unable to answer a multiple-choice question immediately, eliminate the alternatives that you know are incorrect and proceed from that point. The same goes for a multiple-response question that requires you to choose two or more of the given alternatives. If a fill-in-the-blank question poses a problem, read the situation and essential information carefully and then formulate your response. p. 714 A patient is diagnosed with obstructive hypertrophic cardiomyopathy (OHC). The nurse anticipates that which type of drug will be prescribed to relieve the patient's symptoms? <p>A patient is diagnosed with obstructive hypertrophic cardiomyopathy (OHC). The nurse anticipates that which type of drug will be prescribed to relieve the patient's symptoms?</p> Nitrate Antibiotic Vasodilator Beta blocker p. 716 The nurse learns she will be admitting a patient scheduled for a muscle resection with mitral valve replacement. The nurse knows this patient has which type of cardiomyopathy? <p>The nurse learns she will be admitting a patient scheduled for a muscle resection with mitral valve replacement. The nurse knows this patient has which type of cardiomyopathy?</p> Dilated cardiomyopathy Restrictive cardiomyopathy Hypertrophic cardiomyopathy Arrhythmogenic right ventricular cardiomyopathy . pp. 714-715 Which class of medication is administered to a patient with dilated cardiomyopathy to decrease blood pressure and make it easier for the heart to pump blood? <p>Which class of medication is administered to a patient with dilated cardiomyopathy to decrease blood pressure and make it easier for the heart to pump blood?</p> Nitrates Vasodilators Beta blockers Cardiac glycosides p. 716 Which criteria guide candidate selection for heart transplantation? Select all that apply. <p>Which criteria guide candidate selection for heart transplantation? <b>Select all that apply.</b> </p> Age less than 50 years Stable psychosocial status No history of drug or alcohol use Life expectancy of less than 1 year Heart disease for greater than 2 years p. 716 What classifications are considered categories of cardiomyopathies? Select all that apply. <p>What classifications are considered categories of cardiomyopathies? <b>Select all that apply.</b> </p> Dilated Restrictive Ventricular Hypertrophic Structural ventricular Test-Taking Tip: Be alert for details about what you are being asked to do. In this Question Type, you are asked to select all options that apply to a given situation or patient. All options likely relate to the situation, but only some of the options may relate directly to the situation. p. 714 The nurse is providing teaching to a patient who is being discharged after heart transplant surgery. The patient demonstrates a clear understanding of the teaching provided when he identifies which clinical manifestations of heart transplant rejection? Select all that apply. <p>The nurse is providing teaching to a patient who is being discharged after heart transplant surgery. The patient demonstrates a clear understanding of the teaching provided when he identifies which clinical manifestations of heart transplant rejection? <b>Select all that apply.</b> </p> Edema Tachycardia Atrial flutter Hypertension Abdominal bloating p. 716 What findings are common in a patient with dilated cardiomyopathy? Select all that apply. <p>What findings are common in a patient with dilated cardiomyopathy? <b>Select all that apply.</b> </p> Dyspnea Syncope Heart block Mitral valve murmur Pulmonary embolism Test-Taking Tip: Avoid taking a wild guess at an answer. However, should you feel insecure about a question, eliminate the alternatives that you believe are definitely incorrect, and reread the information given to make sure you understand the intent of the question. This approach increases your chances of randomly selecting the correct answer or getting a clearer understanding of what is being asked. p. 715 What findings are common in patients with obstructed hypertrophic cardiomyopathy? Select all that apply. <p>What findings are common in patients with obstructed hypertrophic cardiomyopathy? <b>Select all that apply.</b> </p> Syncope Dyspnea Heart block Mitral valve murmur Pulmonary embolism Test-Taking Tip: Have a general knowledge of which topics are covered and how many questions there are in each topic area. Know the amount of time you will have. Study the sample questions for style and format. p. 715 Which medications should the nurse question for a patient diagnosed with obstructed hypertrophic cardiomyopathy? Select all that apply. <p>Which medications should the nurse question for a patient diagnosed with obstructed hypertrophic cardiomyopathy? <b>Select all that apply.</b> </p> Metoprolol Verapamil Amiodarone Nitroglycerin Isosorbide dinitrate Test-Taking Tip: Avoid looking for an answer pattern or code. There may be times when four or five consecutive questions have the same letter or number for the correct answer. p. 716 A patient with a respiratory rate of 30 breaths per minute has frothy, pink-tinged sputum and nocturia. The nurse hears an S 3 gallop on auscultation. Which condition does the nurse suspect? <p>A patient with a respiratory rate of 30 breaths per minute has frothy, pink-tinged sputum and nocturia. The nurse hears an S <sub>3 </sub>gallop on auscultation. Which condition does the nurse suspect?</p> Mitral stenosis Pulmonary edema Left-sided heart failure Right-sided heart failure p. 695 Which factor reflects the most common etiology of heart failure? <p>Which factor reflects the <b>most</b> common etiology of heart failure?</p> Valvular disorders Ventricular dilation Fluid volume excess Myocardial infarction (MI) Rationale About one-third of patients who have had MI develop heart failure. The second most common cause of heart failure is structural changes such as valvular dysfunction. Fluid volume excess may be a cause or result of heart failure. Ventricular dilation is a result of fluid accumulation from pump failure, but it is not a cause or etiology. p. 694 Which intervention assists the patient with acute pulmonary edema in reducing dyspnea? <p>Which intervention assists the patient with acute pulmonary edema in reducing dyspnea? <span> <br/> </span> </p> Ask a family member to remain with the patient. Monitor pulse oximetry and cardiac rate and rhythm. Place the patient in the high Fowler's position with the legs down. Reassure the patient that the distress can be relieved with proper intervention. Test-Taking Tip: Avoid spending excessive time on any one question. Most questions can be answered in 1 to 2 minutes. p. 703 A patient expectorates frothy, pink-tinged sputum. Which condition does the nurse suspect in the patient? <p>A patient expectorates frothy, pink-tinged sputum. Which condition does the nurse suspect in the patient?</p> Valvular disease Myocardial infarction High-output heart failure Severe pulmonary edema Test-Taking Tip: Many times the correct answer is the longest alternative given, but do not count on it. Item writers (those who write the questions) are also aware of this and attempt to avoid offering you such "helpful hints." p. 695 What does the nurse anticipate administering to a patient with pulmonary edema to promote diuresis? <p>What does the nurse anticipate administering to a patient with pulmonary edema to promote diuresis?</p> Morphine Ultrafiltration Loop diuretic by infusion Nitroglycerin by infusion p. 705 The emergency department nurse is assessing a patient in which heart failure (HF) is suspected. The nurse knows which diagnostic test result is consistent with a diagnosis of HF? <p>The emergency department nurse is assessing a patient in which heart failure (HF) is suspected. The nurse knows which diagnostic test result is consistent with a diagnosis of HF?</p> Ejection fraction of 60% Serum potassium level of 3.2 mEq/L B-type natriuretic peptide (BNP) of 760 ng/dL Chest x-ray report showing right middle lobe consolidation Test-Taking Tip: Make certain that the answer you select is reasonable and obtainable under ordinary circumstances and that the action can be carried out in the given situation. p. 697 The nurse is providing education to a patient with heart failure (HF) based on the Heart Failure Core Measure Set. Which point is essential for the nurse to include? <p>The nurse is providing education to a patient with heart failure (HF) based on the Heart Failure Core Measure Set. Which point is <b>essential</b> for the nurse to include?</p> Encouraging the patient to stop smoking Ensuring oxygen is available for home use Assisting the patient in completing an advance directive Instructing to notify the provider of weight loss greater than 3 lbs in 3 days p. 703 When following up in the clinic with a patient with heart failure, how does the nurse recognize that the patient has been compliant with fluid restrictions? <p>When following up in the clinic with a patient with heart failure, how does the nurse recognize that the patient has been compliant with fluid restrictions?</p> Pedal edema Auscultation of crackles Weight loss of 6 pounds since the last visit Reports sucking on ice chips all day for dry mouth Test-Taking Tip: Do not panic while taking an exam! Panic will only increase your anxiety. Stop for a moment, close your eyes, take a few deep breaths, and resume review of the question. p. 699 The patient presents to the emergency department with pedal edema, crackles on auscultation, and a report of a 10-pound weight gain in 1 week. The nurse suspects heart failure and knows which test will confirm her suspicion? Serum electrolytes Arterial blood gases B-type natriuretic peptide (BNP) Ventilation perfusion (V/Q) scan p. 697 An older adult patient with type 2 diabetes mellitus is being treated with rosiglitazone. To which condition is the patient susceptible? <p>An older adult patient with type 2 diabetes mellitus is being treated with rosiglitazone. To which condition is the patient susceptible?</p> Cardiac infection Difficulty in breathing Obstructive sleep apnea Acute myocardial infarction p. 694 The nurse is caring for a patient with heart failure in the coronary care unit (CCU). The patient is now exhibiting signs of air hunger and anxiety. Which nursing intervention does the nurse perform first for this patient? <p>The nurse is caring for a patient with heart failure in the coronary care unit (CCU). The patient is now exhibiting signs of air hunger and anxiety. Which nursing intervention does the nurse perform <b>first</b> for this patient?</p> Positions the patient to alleviate dyspnea Determines the patient's physical limitations Encourages alternate rest and activity periods Monitors and documents heart rate, rhythm, and pulses Test-Taking Tip: Come to your test prep with a positive attitude about yourself, your nursing knowledge, and your test-taking abilities. A positive attitude is achieved through self-confidence gained by effective study. This means (a) answering questions (assessment), (b) organizing study time (planning), (c) reading and further study (implementation), and (d) answering questions (evaluation). pp. 695, 697 What are common signs and symptoms of right-sided heart failure? Select all that apply. <p>What are common signs and symptoms of right-sided heart failure? <b>Select all that apply.</b> </p> A hacking cough Dependent edema Increase in weight Nausea and vomiting Oliguria during the day p. 695 What finding does the nurse expect when assessing a patient with long-standing heart failure? Select all that apply. <p>What finding does the nurse expect when assessing a patient with long-standing heart failure? <b>Select all that apply.</b> </p> Weight loss Swollen abdomen Third heart sound Shortness of breath with activity Awakening with breathlessness Productive cough with yellow sputum STUDY TIP: The old standbys of enough sleep and adequate nutritional intake also help keep excessive stress at bay. Although nursing students learn about the body's energy needs in anatomy and physiology classes, somehow they tend to forget that glucose is necessary for brain cells to work. Skipping breakfast or lunch or surviving on junk food puts the brain at a disadvantage. p. 695 Before administering furosemide to a patient with heart failure, it is most important for the nurse to assess which diagnostic test result? <p>Before administering furosemide to a patient with heart failure, it is <b>most</b> important for the nurse to assess which diagnostic test result?</p> Serum electrolytes Arterial blood gases B-type natriuretic peptide Hemoglobin and hematocrit Test-Taking Tip: You have at least a 25% chance of selecting the correct response in multiple-choice items. If you are uncertain about a question, eliminate the choices that you believe are wrong and then call on your knowledge, skills, and abilities to choose from the remaining responses. p. 696 What are the symptoms of left-sided heart failure? Select all that apply. <p>What are the symptoms of left-sided heart failure? <b>Select all that apply.</b> </p> Hacking cough Dependent edema Distended abdomen Oliguria during the day Frothy, pink-tinged sputum p. 695 A patient with heart failure is prescribed enalapril. Which instructions are most important for a safe and effective treatment outcome? Select all that apply. <p>A patient with heart failure is prescribed enalapril. Which instructions are <b>most</b> important for a safe and effective treatment outcome? <b>Select all that apply. </b> </p> "Avoid sodium intake." "Check your weight daily." "Avoid potassium-rich foods." "Do not worry if a cough develops." "Move slowly while changing positions." The nurse is providing medication instructions to a patient with heart failure who is prescribed beta blockers and digoxin therapy. Which statement made by the patient indicates a need for correction? <p>The nurse is providing medication instructions to a patient with heart failure who is prescribed beta blockers and digoxin therapy. Which statement made by the patient indicates a need for correction?</p> "I should not take digoxin along with laxatives." "I should take the medication at the same time daily." "I should take two doses at a time if I missed a dose." "I should stop taking the medication only if the primary health care provider advises me to." Test-Taking Tip: Digoxin is a narrow therapeutic drug and beta blocker which has many side effects. Slightest increase in the dose greatly alters serum drug concentration levels that may adversely affect the patient. p. 698 The patient has heart failure. Which signs or symptoms would prompt the nurse to suspect pulmonary edema? Select all that apply. <p>The patient has heart failure. Which signs or symptoms would prompt the nurse to suspect pulmonary edema? <b>Select all that apply.</b> </p> Increased urination Decreased heart rate Crackles in the lung bases Difficulty in breathing at rest Disorientation regarding time and place p. 702 The nurse is caring for a patient receiving digoxin therapy. Which finding would indicate to the nurse that the patient is suffering from digoxin toxicity? <p>The nurse is caring for a patient receiving digoxin therapy. Which finding would indicate to the nurse that the patient is suffering from digoxin toxicity?</p> Weight gain Blurred vision Worsening heart failure Orthostatic hypotension Rationale Symptoms of digoxin toxicity include blurred vision, fatigue, bradycardia, dysrhythmia and ectopic beats, and changes in mental status. Digoxin toxicity does not specifically worsen heart failure (HF); however, bradycardia may occur. Weight gain occurs when excess sodium is consumed or HF is worsening. Orthostatic hypotension is caused by vasodilating medications, not inotropic agents. p. 700 The nurse is caring for an 82-year-old patient admitted for exacerbation of heart failure (HF). The nurse questions the patient about the use of which medication because it raises an index of suspicion as to the cause of the worsening of the patient's HF? <p>The nurse is caring for an 82-year-old patient admitted for exacerbation of heart failure (HF). The nurse questions the patient about the use of which medication because it raises an index of suspicion as to the cause of the worsening of the patient's HF?</p> Ibuprofen NPH insulin Levothyroxine Hydrochlorothiazide p. 690 While caring for a patient receiving digoxin for heart failure, the nurse becomes concerned when the patient demonstrates which sign/symptom? <p>While caring for a patient receiving digoxin for heart failure, the nurse becomes concerned when the patient demonstrates which sign/symptom?</p> Weight loss Increased urine output Dry tongue and tented skin Sinus bradycardia, rate of 48 beats/min Test-Taking Tip: Identify option components as correct or incorrect. This may help you identify a wrong answer. Example: If you are being asked to identify a diet that is specific to a certain condition, your knowledge about that condition would help you choose the correct response (e.g., cholecystectomy = low-fat, high-protein, low-calorie diet). p. 700 The nurse caring for a patient with heart failure is concerned that digoxin toxicity has developed. For which signs and symptoms of digoxin toxicity does the nurse notify the provider? Select all that apply. <p>The nurse caring for a patient with heart failure is concerned that digoxin toxicity has developed. For which signs and symptoms of digoxin toxicity does the nurse notify the provider? <b>Select all that apply.</b> </p> Fatigue Anorexia Hypokalemia Sinus bradycardia Serum digoxin level of 1.5 Test-Taking Tip: Avoid looking for an answer pattern or code. There may be times when four or five consecutive questions have the same letter or number for the correct answer. p. 700 The nurse prepares to administer digoxin to a patient with heart failure and notes the following information: temperature: 99.8° F; pulse: 48; irregular respirations: 20; potassium level: 3.2 mEq/L. What action does the nurse take? <p>The nurse prepares to administer digoxin to a patient with heart failure and notes the following information: temperature: 99.8&#xb0; F; pulse: 48; irregular respirations: 20; potassium level: 3.2 mEq/L. What action does the nurse take?</p> Give digoxin; reassess the heart rate in 30 minutes Give the digoxin; document assessment findings in the medical record Hold the digoxin and obtain a prescription for a potassium supplement Hold the digoxin and obtain a prescription for an additional dose of furosemide Test-Taking Tip: Pace yourself while taking a quiz or exam. Read the entire question and all answer choices before answering the question. Do not assume that you know what the question is asking without reading it entirely. p. 700 The use of which drug may pose a risk for the development of heart failure, especially in older adults? <p>The use of which drug may pose a risk for the development of heart failure, especially in older adults?</p> Insulin Digoxin Ibuprofen Potassium chloride p. 694 Which factor is the most common etiology of heart failure? <p>Which factor is the <b>most</b> common etiology of heart failure?</p> Obesity Hypertension Hyperkinetic conditions Structural heart changes p. 694 The nurse is providing education to a patient with heart failure (HF) about an appropriate diet. Which information is most important for the nurse to emphasize? <p>The nurse is providing education to a patient with heart failure (HF) about an appropriate diet. Which information is <b>most</b> important for the nurse to emphasize?</p> "Avoid grapefruit juice." "Do not add any salt to your food." "Increase fluids to 100 ounces daily." "Try to increase the amount of green leafy vegetables you eat." Test-Taking Tip: Key words or phrases in the stem of the question such as first, primary, early, or best are important. Similarly, words such as only, always, never, and all in the alternatives are frequently evidence of a wrong response. As in life, no real absolutes exist in nursing; however, every rule has its exceptions, so answer with care. p. 694 Which assessment finding is an indication of left-sided heart failure? <p>Which assessment finding is an indication of left-sided heart failure?</p> Nocturia Peripheral edema Swollen abdomen Crackles in the lung fields Test-Taking Tip: Because few things in life are absolute without exceptions, avoid selecting answers that include words such as always, never, all, every, and none. Answers containing these key words are rarely correct. p. 695 The nurse is caring for a patient with heart failure. For which symptoms does the nurse assess? Select all that apply. <p>The nurse is caring for a patient with heart failure. For which symptoms does the nurse assess? <b>Select all that apply.</b> </p> Fatigue Tachycardia Chest discomfort or pain Sleeps on back without a pillow Expectorates thick, yellow sputum p. 695 The nurse is providing community education on the risks for heart failure. Which topics does the nurse include in the discussion? Select all that apply. Cocaine use Hypotension Aortic stenosis Hypothyroidism Sinus arrhythmia Use of anxiolytic medications STUDY TIP: Laughter is a great stress reliever. Watching a short program that makes you laugh, reading something funny, or sharing humor with friends helps decrease stress. p. 694 The nurse is providing discharge teaching to a patient with heart failure, focusing on when to seek medical attention. Which statement by the patient indicates a correct understanding of the teaching? <p>The nurse is providing discharge teaching to a patient with heart failure, focusing on when to seek medical attention. Which statement by the patient indicates a correct understanding of the teaching?</p> "I should expect occasional chest pain." "I will try walking for 1 hour each day." "I will report to the provider weight loss of 2-3 pounds in a day." "I will call the provider if I have a cough lasting 3 or more days." Test-Taking Tip: As you answer each question, write a few words about why you think that answer is correct; in other words, justify why you selected that answer. If an answer you provide is a guess, mark the question to identify it. This will permit you to recognize areas that need further review. It will also help you to see how correct your "guessing" can be. Remember: on the licensure examination you must answer each question before moving on to the next question. p. 704 Which diagnostic reading does the nurse anticipate may be elevated in a patient with heart failure? <p>Which diagnostic reading does the nurse anticipate may be elevated in a patient with heart failure?</p> Sodium Calcium Cardiac output B-type natriuretic peptide p. 694 After receiving a change-of-shift report about these four patients, which patient should the nurse assess first? <p>After receiving a change-of-shift report about these four patients, which patient should the nurse assess <b>first</b>?</p> 68-year-old with pericarditis who is reporting sharp, stabbing chest pain when taking deep breaths 55-year-old admitted with pulmonary edema who received furosemide and whose current O 2saturation is 94% 46-year-old with aortic stenosis who takes digoxin and has new-onset frequent premature ventricular contractions 79-year-old admitted for possible rejection of a heart transplant who has sinus tachycardia, heart rate 104 beats/min Rationale The 46-year-old's premature ventricular contractions may be indicative of digoxin toxicity; further assessment for clinical manifestations of digoxin toxicity should be done and the health care provider notified about the dysrhythmia. The 55-year-old is stable and can be assessed after the patient with aortic stenosis. The 68-year-old may be assessed after the patient with aortic stenosis; this type of pain is expected in pericarditis. Tachycardia is expected in the 79-year-old because rejection will cause signs of decreased cardiac output, including tachycardia; this patient may be seen after the patient with aortic stenosis. STUDY TIP: The old standbys of enough sleep and adequate nutritional intake also help keep excessive stress at bay. Although nursing students learn about the body's energy needs in anatomy and physiology classes, somehow they tend to forget that glucose is necessary for brain cells to work. Skipping breakfast or lunch or surviving on junk food puts the brain at a disadvantage. p. 700 Which nursing action should the nurse delegate to a nursing assistant working on the medical unit? <p>Which nursing action should the nurse delegate to a nursing assistant working on the medical unit?</p> Check for peripheral edema in a patient with endocarditis Monitor the pain level for a patient with acute pericarditis Obtain daily weights for several patients with class IV heart failure Determine the usual alcohol intake for a patient with cardiomyopathy STUDY TIP: Becoming a nursing student automatically increases stress levels because of the complexity of the information to be learned and applied and because of new constraints on time. One way to decrease stress associated with school is to become very organized so that assignment deadlines or tests do not come as sudden surprises. By following a consistent plan for studying and completing assignments, you can stay on top of requirements and thereby prevent added stress. p. 696 The home health nurse visits a patient with heart failure who has gained 5 pounds in the past 3 days. The patient states, "I feel so tired and short of breath." Which action does the nurse take first? Assess the patient for peripheral edema Listen to the patient's posterior breath sounds Remind the patient about dietary sodium restrictions Notify the health care provider about the patient's weight gain Rationale Because the patient is at risk for pulmonary edema and hypoxemia, the first action should be to assess breath sounds. Assessment of edema may be delayed until after breath sounds are assessed. After a full assessment, the nurse should notify the health care provider. After physiologic stability is attained, then ask the patient about behaviors that may have caused the weight gain, such as increased sodium intake or changes in medications. Test-Taking Tip: Be aware that information from previously asked questions may help you respond to other examination questions. p. 696 How does the nurse in the cardiac clinic recognize that the patient with heart failure has demonstrated a positive outcome related to the addition of metoprolol to the medication regimen? <p>How does the nurse in the cardiac clinic recognize that the patient with heart failure has demonstrated a positive outcome related to the addition of metoprolol to the medication regimen?</p> Ejection fraction is 25% Patient reports that she experiences palpitations Patient states that she is able to sleep on one pillow Patient was hospitalized five times last year with pulmonary edema Test-Taking Tip: The computerized NCLEX exam is an individualized testing experience in which the computer chooses your next question based on the ability and competency you have demonstrated on previous questions. The minimum number of questions will be 75 and the maximum 265. You must answer each question before the computer will present the next question, and you cannot go back to any previously answered questions. Remember that you do not have to answer all of the questions correctly to pass. p. 700 The nurse discusses the importance of restricting sodium in the diet of a patient with heart failure. Which statement made by the patient indicates that the patient needs further teaching? <p>The nurse discusses the importance of restricting sodium in the diet of a patient with heart failure. Which statement made by the patient indicates that the patient needs further teaching?</p> "I should avoid eating hamburgers." "I must cut out bacon and canned foods." "I shouldn't put the salt shaker on the table anymore." "I should avoid lunchmeats but may cook my own turkey." Test-Taking Tip: Become familiar with reading questions on a computer screen. Familiarity reduces anxiety and decreases errors. pp. 703, 705 A patient reports waking with a feeling of breathlessness 2 to 5 hours after falling asleep. What condition may this indicate? <p>A patient reports waking with a feeling of breathlessness 2 to 5 hours after falling asleep. What condition may this indicate?</p> Orthopnea Exertional dyspnea High-output heart failure Paroxysmal nocturnal dyspnea STUDY TIP: Determine whether you are a "lark" or an "owl." Larks, day people, do best getting up early and studying during daylight hours. Owls, night people, are more alert after dark and can remain up late at night studying, catching up on needed sleep during daylight hours. It is better to work with natural biorhythms than to try to conform to an arbitrary schedule. You will absorb material more quickly and retain it better if you use your most alert periods of each day for study. Of course, it is necessary to work around class and clinical schedules. Owls should attempt to register in afternoon or evening lectures and clinical sections; larks do better with morning lectures and day clinical sections. p. 695 Which specific type of heart failure (HF) is most likely to occur at the beginning of chronic HF? <p>Which specific type of heart failure (HF) is <b>most</b> likely to occur at the beginning of chronic HF?</p> Bilateral Left-sided Right-sided High-output Rationale Most HF begins with failure of the left ventricle and progresses to failure of both ventricles. Right-sided HF may be caused by left ventricular failure. High-output failure is an uncommon type of HF where cardiac output can be normal or above-normal and is caused by increased metabolic needs or hyperkinetic conditions. Bilateral HF usually occurs after left-sided HF. p. 695 Which intervention does the nurse perform to decrease dyspnea in a patient with acute heart failure? <p>Which intervention does the nurse perform to decrease dyspnea in a patient with acute heart failure?</p> Avoid waking the patient at night Elevate swollen legs above heart level Place the patient in high Fowler's position Assess for orthostatic hypotension and dizziness Test-Taking Tip: Be alert for details. Details provided in the stem of the item, such as behavioral changes or clinical changes (or both) within a certain time period, can provide a clue to the most appropriate response or, in some cases, responses. p. 703 Which are major compensatory mechanisms when cardiac output is insufficient to meet the demands of the body? Select all that apply. <p>Which are major compensatory mechanisms when cardiac output is insufficient to meet the demands of the body? <b>Select all that apply.</b> </p> Chemical responses Myocardial hypertrophy Pulmonary hypertension Renin-angiotensin system activation Sympathetic nervous system stimulation p. 692 A patient admitted for heart failure has a priority problem of hypervolemia related to compromised regulatory mechanisms. Which assessment data obtained the day after admission is the best indicator that the treatment has been effective? <p>A patient admitted for heart failure has a priority problem of hypervolemia related to compromised regulatory mechanisms. Which assessment data obtained the day after admission is the <b>best</b> indicator that the treatment has been effective?</p> The patient's weight decreases by 2.5 kg The patient's blood pressure is 122/84 mm Hg The patient has diuresis of 400 mL in 24 hours The patient has an apical pulse of 82 beats/min Test-Taking Tip: The night before the examination you may wish to review some key concepts that you believe need additional time, but then relax and get a good night's sleep. Remember to set your alarm, allowing yourself plenty of time to dress comfortably (preferably in layers, depending on the weather), have a good breakfast, and arrive at the testing site at least 15 to 30 minutes early. p. 695 In testing arterial blood gas values, which condition can be a probable cause of metabolic acidosis? <p>In testing arterial blood gas values, which condition can be a probable cause of metabolic acidosis?</p> Hyperventilation Potassium retention Carbon dioxide retention Accumulation of lactic acid STUDY TIP: Rest is essential to the body and brain for good performance; think of it as recharging the battery. A run-down battery provides only substandard performance. For most students, it is better to spend 7 hours sleeping and 3 hours studying than to cut sleep to 6 hours and study 4 hours. The improvement in the rested mind's efficiency will balance out the difference in the time spent studying. Knowing your natural body rhythms is necessary when it comes to determining the amount of sleep needed for personal learning efficiency. p. 697 Which nursing intervention for a patient admitted today with heart failure will assist the patient to conserve energy? <p>Which nursing intervention for a patient admitted today with heart failure will assist the patient to conserve energy?</p> Patient ambulates around the nursing unit with a walker The nurse returns the patient to bed when he becomes tachycardic The nurse monitors the patient's pulse and blood pressure frequently The nurse obtains a bedside commode before administering furosemide Test-Taking Tip: Try putting questions and answers in your own words to test your understanding. p. 710 Which ventricular reconstructive procedure involves the removal of portions of the cardiac septum and left ventricular wall? <p>Which ventricular reconstructive procedure involves the removal of portions of the cardiac septum and left ventricular wall?</p> Myosplint Partial left ventriculectomy Acorn cardiac support device Endoventricular circular patch cardioplasty p. 702 One hour after administering enalapril to a patient with heart failure (HF), the patient reports dizziness and the nurse notes a blood pressure of 82/46 mm Hg. Which is the nurse's priority action? <p>One hour after administering enalapril to a patient with heart failure (HF), the patient reports dizziness and the nurse notes a blood pressure of 82/46 mm Hg. Which is the nurse's <b>priority</b> action?</p> Initiate cardiac monitoring Place the patient flat in bed Obtain an order for dopamine Notify the health care provider Test-Taking Tip: Be alert for grammatical inconsistencies. If the response is intended to complete the stem (an incomplete sentence) but makes no grammatical sense to you, it might be a distractor rather than the correct response. Question writers typically try to eliminate these inconsistencies. p. 685 A patient has refractory end-stage heart failure. In which class of the New York Heart Association (NYHA) functional classification system is the patient categorized? <p>A patient has refractory end-stage heart failure. In which class of the New York Heart Association (NYHA) functional classification system is the patient categorized?</p> Class I NYHA Class II NYHA Class III NYHA Class IV NYHA p. 692 Which parameter, determined through laboratory testing, is known as the "early warning detector" for heart failure? <p>Which parameter, determined through laboratory testing, is known as the "early warning detector" for heart failure?</p> Hematocrit Microalbuminuria Electrocardiogram B-type natriuretic peptide p. 683 A patient has a history of acute dyspnea. Which test is used for diagnosing diastolic heart failure in this patient? <p>A patient has a history of acute dyspnea. Which test is used for diagnosing diastolic heart failure in this patient?</p> Chest x-rays Electrocardiogram Multigated angiographic B-type natriuretic peptide p. 683 What are common symptoms of hypokalemia? Select all that apply. <p>What are common symptoms of <i>hypokalemia</i>? <b>Select all that apply.</b> </p> Weakness Cool extremities Dependent edema Depressed reflexes Irregular heart rate p. 699 A patient with heart failure is taking furosemide. Which finding concerns the nurse with this new prescription? <p>A patient with heart failure is taking furosemide. Which finding concerns the nurse with this new prescription?</p> Serum creatinine of 1.0 mg/dL Serum sodium level of 135 mEq/L Serum potassium level of 2.8 mEq/L Serum magnesium level of 1.9 mEq/L Test-Taking Tip: Look for answers that focus on the patient or that are directed toward the patient's feelings. p. 686 The nurse in the emergency department is caring for a patient with acute heart failure who is experiencing severe dyspnea; pink, frothy sputum; and crackles throughout the lung fields. Which order would the nurse implement first? <p>The nurse in the emergency department is caring for a patient with acute heart failure who is experiencing severe dyspnea; pink, frothy sputum; and crackles throughout the lung fields. Which order would the nurse implement <b>first</b>? <img src="https://eolscontent.elsevier.com/1069FQX38FT/image/2011_iggy_ch35_fig1.png" alt="2011_iggy_ch35_fig1.png"/> </p> Administer subcutaneous heparin Administer oral enalapril Administer intravenous furosemide STUDY TIP: Record the information you find to be most difficult to remember on 3" × 5" cards and carry them with you in your pocket or purse. When you are waiting in traffic or for an appointment, just pull out the cards and review again. This "found" time may add points to your test scores that you have lost in the past. p. 698 A patient who has been admitted for the third time this year for heart failure says, "This isn't worth it anymore. I just want it all to end." What is the nurse's best response? <p>A patient who has been admitted for the third time this year for heart failure says, "This isn't worth it anymore. I just want it all to end." What is the nurse's <b>best</b> response?</p> Tells the patient that things will get better Calls the family to lift the patient's spirits Considers further assessment for depression Sedates the patient to decrease myocardial oxygen demand p. 696 The nurse teaches a patient with heart failure to begin exercising when permitted by the health care provider. Which beginning activity does the nurse suggest as appropriate? <p>The nurse teaches a patient with heart failure to begin exercising when permitted by the health care provider. Which beginning activity does the nurse suggest as appropriate?</p> Jogging Knitting Walking Weightlifting p. 704 For which heart condition is it most important for the home health nurse to discuss an advance directive with the patient and family? <p>For which heart condition is it <b>most</b> important for the home health nurse to discuss an advance directive with the patient and family?</p> Heart failure Rheumatic carditis Infective endocarditis Valvular heart disease p. 705 What are the intended outcomes of ultrafiltration when performed in a patient with renal dysfunction? Select all that apply. <p>What are the intended outcomes of ultrafiltration when performed in a patient with renal dysfunction? <b>Select all that apply.</b> </p> Increase in aldosterone Decrease in cardiac index Decrease in filling pressures Reduction in norepinephrine Decrease in pulmonary arterial pressure p. 703 What are symptoms of aortic stenosis? Select all that apply. <p>What are symptoms of aortic stenosis? <b>Select all that apply.</b> </p> Polyuria Debilitation Marked fatigue Nocturnal angina Peripheral cyanosis p. 707 Which conditions can cause high-output heart failure? Select all that apply. <p>Which conditions can cause high-output heart failure? <b>Select all that apply.</b> </p> High fever Septicemia Valvular disease Hyperthyroidism Coronary artery disease p. 692 Which patients belong to Category B according to the American College of Cardiology (ACC) and the American Heart Association (AHA)? <p>Which patients belong to Category B according to the American College of Cardiology (ACC) and the American Heart Association (AHA)?</p> Patients with refractory end-stage heart failure Patients at high risk for developing heart failure Patients with present or past symptoms of heart failure Patients with cardiac structural abnormalities without symptoms of heart failure p. 692 Which of the nurse's four patients does the nurse anticipate has left-sided heart failure associated with decreased cardiac output? <p>Which of the nurse's four patients does the nurse anticipate has left-sided heart failure associated with decreased cardiac output? <img src="https://eolscontent.elsevier.com/1069FQX38FT/image/2006_iggy_ch35_fig2.png" alt="2006_iggy_ch35_fig2.png"/> </p> Patient 1 Patient 2 Patient 3 Patient 4 Test-Taking Tip: Chart/exhibit items present a situation and ask a question. A variety of objective and subjective information is presented about the patient in formats such as the medical record (e.g., laboratory test results, results of diagnostic procedures, progress notes, health care provider orders, medication administration record, health history), physical assessment data, and assistant/patient interactions. After analyzing the information presented, the test taker answers the question. These questions usually reflect the analyzing level of cognitive thinking. p. 695 An older patient undergoing treatment of chronic heart failure (HF) with atrial fibrillation reports anorexia, fatigue, blurred vision, and changes in mental status. Which medication does the nurse suspect to be the cause of the patient's condition? <p>An older patient undergoing treatment of chronic heart failure (HF) with atrial fibrillation reports anorexia, fatigue, blurred vision, and changes in mental status. Which medication does the nurse suspect to be the cause of the patient's condition?</p> Digoxin Carvedilol Dobutamine Levosimendan p. 700 While assessing for signs of heart failure in a patient, the nurse notes that the patient has poor tissue perfusion. Which other symptoms would have been observed in the patient? Select all that apply. <p>While assessing for signs of heart failure in a patient, the nurse notes that the patient has poor tissue perfusion. Which other symptoms would have been observed in the patient? <b>Select all that apply.</b> </p> Fatigue Cyanosis Weight gain Cool extremities Jugular venous distention pp. 695-696 A registered nurse is teaching a nursing student about mitral valve stenosis and mitral valve regurgitation. The nursing student demonstrates understanding of the teaching by identifying which common effect of both mitral valve disorders? Fatigue Hemoptysis Hepatomegaly High-pitched holosystolic murmur The nurse notes the patient who recently underwent heart transplant surgery is becoming increasingly fatigued. Which symptoms, in addition to the fatigue, support the nurse's concern that a patient's body has rejected a heart transplant? Select all that apply. <p>The nurse notes the patient who recently underwent heart transplant surgery is becoming increasingly fatigued. Which symptoms, in addition to the fatigue, support the nurse's concern that a patient's body has rejected a heart transplant? <b>Select all that apply.</b> </p> Hypertension New bradycardia Decreased weight Abdominal bloating Increased ejection fraction Test-Taking Tip: Be alert for details about what you are being asked to do. In this Question Type, you are asked to select all options that apply to a given situation or patient. All options likely relate to the situation, but only some of the options may relate directly to the situation. p. 716 What teaching intervention is the nurse least likely to provide to a stable chronic heart failure patient? <p>What teaching intervention is the nurse <b>least</b> likely to provide to a stable chronic heart failure patient?</p> "You should avoid overdoing exercise." "You should start exercising only when your edema stabilizes." "You need to avoid salty food and addition of table salt to food." "You cannot claim Medicare reimbursement for cardiac rehabilitation." p. 704 According to the American College of Cardiology (ACC) and the American Heart Association (AHA), which patients does category D include? <p>According to the American College of Cardiology (ACC) and the American Heart Association (AHA), which patients does category D include?</p> Patients with end-stage heart failure Patients at high risk for developing heart failure conditions Patients with current or previous symptoms of heart failure Patients with cardiac structural abnormalities or remodeling p. 692 Which category does an individual with current or prior symptoms of heart failure belong to according to the American College of Cardiology (ACC) and the American Heart Association (AHA)? <p>Which category does an individual with current or prior symptoms of heart failure belong to according to the American College of Cardiology (ACC) and the American Heart Association (AHA)?</p> A B C D p. 692 Which objective listed by a nursing student about older adults with cardiac disease from the Healthy People 2020 initiative indicates a need for further teaching by the precepting nurse? <p>Which objective listed by a nursing student about older adults with cardiac disease from the <i>Healthy People 2020</i> initiative indicates a need for further teaching by the precepting nurse?</p> Ensure that the interdisciplinary team provides follow-up care in the home or nursing home Call the hospitalized patient to check that he or she has no signs or symptoms of impending heart failure Provide a continuing plan of care for the patient and family or other caregivers when the patient is discharged from the hospital Collaborate with the case manager for discharge planning, including adequate support in the community, of patients hospitalized for heart failure p. 695 Which teaching will the nurse provide for the patient with valvular heart disease? <p>Which teaching will the nurse provide for the patient with valvular heart disease?</p> "Notify only your cardiologist about a defective heart valve." "Tell your dentist about the disease before having invasive dental work." "Take antibiotics after a procedure only when they have been prescribed." "Call you primary health care provider for a fever that lasts for three or more days." p. 712 A patient begins lisinopril therapy. What does the nurse consider at the start of therapy with this medication? <p>A patient begins lisinopril therapy. What does the nurse consider at the start of therapy with this medication?</p> Liver function tests The risk for hypotension The potential for bradycardia The patient's ability to understand medication teaching [Show More]

Last updated: 1 year ago

Preview 10 out of 180 pages

Buy Now

Instant download

We Accept:

We Accept
loader

Loading document previews ...

Buy this document to get the full access instantly

Instant Download Access after purchase

Buy Now

Instant download

We Accept:

We Accept

Reviews( 0 )

$14.00

Buy Now

We Accept:

We Accept

Instant download

Can't find what you want? Try our AI powered Search

425
0

Document information


Connected school, study & course


About the document


Uploaded On

Jun 24, 2020

Number of pages

180

Written in

Seller


seller-icon
Kirsch

Member since 5 years

941 Documents Sold

Reviews Received
111
37
8
4
28
Additional information

This document has been written for:

Uploaded

Jun 24, 2020

Downloads

 0

Views

 425

Document Keyword Tags


$14.00
What is Scholarfriends

In Scholarfriends, a student can earn by offering help to other student. Students can help other students with materials by upploading their notes and earn money.

We are here to help

We're available through e-mail, Twitter, Facebook, and live chat.
 FAQ
 Questions? Leave a message!

Follow us on
 Twitter

Copyright © Scholarfriends · High quality services·